Superpage
Page views in 2024 to date: 10

Anatomy & thymus histology
Definition / general
  • Mediastinum is the central portion of the thoracic cavity (between the pleural cavities, from sternum to spine and thoracic inlet to diaphragm), which contains the heart, great vessels, esophagus, trachea and thymus as well many other vessels and neural components; it can be divided into 4 regions
  • Thymus is a partially lobulated, encapsulated, primary lymphoid organ that plays a central role in the development of cell mediated immunity and T cell differentiation
Essential features
  • Mediastinum is the centrally located portion of thorax, which notably contains the heart, aorta, vena cava, esophagus, trachea, thymus, thoracic duct, sympathetic trunk, phrenic and vagus nerves, internal thoracic arteries, azygos and hemiazygos veins
  • Mediastinum can be divided into 4 regions: superior, anterior, middle and posterior
  • Thymus is a partially lobulated, encapsulated, primary lymphoid organ that plays a central role in the development of cell mediated immunity and T cell differentiation
  • Thymus is active before birth and in childhood; after puberty the cortex involutes and is replaced by adipose tissue
Terminology
  • Mediastinal cavity
Gross anatomy - mediastinum
  • Mediastinum is the anatomical compartment that runs the length of the thoracic cavity (longitudinally from the thoracic inlet to the diaphragm) and comprises the area between the lungs / pleura; it is further divided into 4 compartments
    • Superior mediastinum
      • Area inferior to the thoracic outlet (~first rib) and superior to the thoracic plane (sternal angle ventrally and the T4 - T5 junction dorsally)
      • Contains the aortic arch (including brachiocephalic trunk, left common carotid and left subclavian arteries), esophagus, trachea, thymus, phrenic and vagus nerves
    • Anterior mediastinum
      • Area that is ventral to anterior cardiac border and aortic root and dorsal to the body of the sternum
      • Contains the internal thoracic arteries and transversus thoracis muscles
    • Middle mediastinum
      • Area inferior to the thoracic plane, medial to the pleural sacs and superior to the diaphragm
      • Contains the heart, ascending aorta and superior vena cava, pulmonary trunk and veins, trachea and bronchi
    • Posterior mediastinum
      • Area ventral to the posterior thoracic body wall and dorsal to the posterior surface of the heart (base)
      • Contains the descending aorta, esophagus, thoracic duct, sympathetic trunk, azygos and hemiazygos veins
  • References: Thorac Surg Clin 2011;21:251, Ann Ital Chir 2007;78:351, StatPearls: Anatomy, Thorax, Heart and Pericardial Cavity [Accessed 26 February 2024], StatPearls: Anatomy, Thorax, Mediastinum [Accessed 26 February 2024], Mediastinum 2023;7:14
Embryology and anatomy - thymus
  • Thymus arises from the endoderm of the third and fourth branchial clefts (Br Med J 1963;2:459)
  • At week 6, it develops as 2 separate lobes and at week 8, the lobes migrate to fuse in the anterior superior mediastinum; however, the blood supply, lymphatic drainage and innervation of each lobe remain separate
  • At week 10, lobules begin to form as a result of the migration of hematopoietic cells from the liver
  • Differentiation into cortex and medulla occurs at 14 - 16 weeks
  • Thymus reaches maturity in utero
    • Ratio of thymus to body weight is its greatest before birth; however, the thymus reaches its maximum weight just before puberty, before undergoing involution
  • Mature thymus
    • Located in the anterior superior mediastinum, retrosternally
    • Bilobed organ with a cortex and medulla
    • It is supplied with blood via the internal thoracic and inferior thyroid arteries
    • Has no afferent lymphatic vessels
    • Has a blood thymus barrier that prevents the transport of macromolecules from the vasculature into the thymic cortical parenchyma
  • Involution of the thymus involves its cellular structure being replaced with adipose tissue; this also is related to a decrease in function
  • Thymic remnants may be present in adult prepericardial or retrocarinal fat
  • References: StatPearls: Anatomy, Head and Neck - Blood Thymus Barrier [Accessed 26 February 2024], Radiographics 2006;26:335
Physiology - thymus
  • Primary organ responsible for the production and maturation of lymphocytes and development of the immune system (Adv Clin Exp Med 2016;25:369)
    • It has an important role in development of cell mediated immunity and T cell differentiation
    • Positive selection of self antigen recognizing T cells induces apoptosis; surviving T cells do not react with self antigen
    • Negative selection of lymphocytes that have high affinity binding with the antigen; these also undergo apoptosis
    • Lymphocytes that pass both positive and negative selection can travel outside the thymus, where they are activated by bacteria, viruses or other foreign antigens
    • In thymus, 95% of all T cells undergo apoptosis due to their ability to recognize self antigen presented by the major histocompatibility complex (MHC) system
    • Clonal selection / activation of only those cells that can recognize antigen; this leads to secretion of antibodies, mitosis of T cells and a resulting increase in T cells that recognize the antigen to a pathogen
    • Once the pathogen is eliminated, most of the activated cells undergo apoptosis, with a few surviving as memory cells
  • Thymic involution (Geroscience 2021;43:1369, Adv Clin Exp Med 2016;25:369, Z Gerontol Geriatr 2000;33:341)
    • Normal age related loss of organized architecture with increased adipose cell deposition, resulting in a replacement of functional cells with fat throughout the organ
    • Scarcity of T cell precursors from bone marrow and effects of circulating cytokines and hormones (e.g., hypothesized sex hormone dependent involution) contribute to involution
    • Involution is thought to contribute to immunosenescence, which increases susceptibility to neoplasia, infection and autoimmune diseases
  • Undernutrition / malnutrition can affect thymic microenvironment and immune function (Front Nutr 2022;9:948488)
  • Early programming of thymus, sexual dimorphism, specific T cell progenitors and thymic microenvironment may influence long term immunity and determine progression of involution at older ages (Aging Dis 2012;3:280)
  • Thymocytes in people > 60 years old have higher levels of Ki67 and p53 (Bull Exp Biol Med 2011;151:460)
  • ABH histoblood group antigens are heterogenically expressed in the epithelial cells and lymphocytes of normal thymus and may become immunoreactive in myasthenia gravis and thymoma (APMIS 2006;114:669)
Diagrams / tables

Images hosted on other servers:
Mediastinum anatomy, sagittal

Mediastinum anatomy, sagittal

Mediastinum anatomy, transverse

Mediastinum anatomy, transverse

Histological structure of thymus

Histological structure of thymus

Thymus, hyperplasia, thymoma

Thymus, hyperplasia, thymoma

Clinical features
  • Superior mediastinum: lymphoma, thyroid lesions, parathyroid adenoma
    • Thymus: thymomas, thymic cyst
    • Myasthenia gravis: thymic hyperplasia, thymoma
  • Anterior mediastinum: thymic epithelial tumors and cysts, substernal thyroid, germ cell neoplasms, lymphoproliferative lesions, lymphoma, retrosternal thyroid glandular proliferations, parathyroid lesions, aorticopulmonary type paragangliomas, lymphangioma, hemangioma, lipoma (JBR-BTR 2012;95:281)
  • Middle mediastinum: aneurysm, pleuropericardial cyst, bronchogenic cyst, diaphragmatic hernia, lymphadenopathy
  • Posterior mediastinum: neurogenic masses (schwannoma, neurofibroma, ganglioneuroma, ganglioneuroblastoma, malignant peripheral nerve sheath tumor, neuroblastoma, paraganglioma), meningocele, lymphoma, esophageal disease, aneurysm
  • Superior vena cava syndrome: partial blockage or compression of the superior vena cava; affects superior and middle mediastinum
  • Mediastinal fibrosis / sclerosing mediastinitis: benign, progressive proliferation of fibrous tissue within the mediastinum (BJR Case Rep 2016;2:20150274)
Laboratory
  • Percutaneous needle biopsy with imaging guidance allows for extrapleural access (Radiographics 2005;25:763)
    • Parasternal approach for anterior or middle mediastinal lesions
    • Paravertebral approach for subcarinal and posterior mediastinal lesions
    • Transsternal approach for anterior or middle lesions that are not accessible by the parasternal approach
    • Suprasternal approach for superior mediastinal lesions
  • Ultrasound guided transthoracic biopsy has been shown to be a viable approach in countries without ready access to video assisted procedures (Cureus 2021;13:e13914)
  • Diagnosis of thymoma is usually via clinical and radiological findings; laboratory blood tests are not indicated
    • However, laboratory studies may be helpful in those suspected to have myasthenia gravis or another autoimmune condition or to rule out germ cell tumor
Gross description - thymus
  • Thymus is a partially lobulated lymphoid organ with connective tissue capsule, cortex and medulla; its parenchyma is made of T cells and stroma is made of epithelial reticular and other cells (StatPearls: Anatomy, Head and Neck, Thymus [Accessed 26 February 2024])
    • Cortex: densely packed T cells, basophilic, early T cell development, partially lobulated by connective tissue septa
    • Medulla: Hassall corpuscles, T cell later development, continuous appearance and lighter staining on H&E
  • Thymus has no nodules (B cells), no hilus (only efferent lymphatics) and no subcapsular sinus
  • Mature adult thymic cortex is mostly adipose tissue, where most of the lymphocytes have been replaced by fat; in this case, the most prominent features are Hassall corpuscles
Gross images

Images hosted on other servers:
Morphology of thymus

Morphology of thymus

Adult persistent thymus, cadaver

Adult persistent thymus, cadaver

Normal thymus, 29 weeks gestation

Normal thymus, 29 weeks gestation

Frozen section description
  • In one study, intraoperative frozen section of mediastinal neoplasm without preoperative cytological or histological diagnosis resulted in the correct diagnosis of benign lesions 91% of the time (Eur J Cardiothorac Surg 1990;4:584)
    • However, in 35.5% of cases, intraoperative classification was not possible and in 27.6% of cases, the diagnosis was wrong, resulting in surgical overtreatment for lymphoma in 3 cases
Microscopic (histologic) description
  • Thymus has a connective tissue capsule with septa that extend into the organ, forming partial lobules
  • Lymphocyte nuclei are numerous in the cortex, giving it a generally darker microscopic appearance (on H&E) and a lighter appearing medulla
  • Parenchyma consists of T cells (of varying phenotypes) and B cells within the medulla and perivascular space (which increase with age) (Hum Pathol 2001;32:926)
  • Thymus of children ages 3 - 6 seem to be the most active, demonstrating high numbers of total thymocytes and many CD34+ progenitors (J Allergy Clin Immunol 2005;115:834)
  • Thymic stroma consists of all of the nonhematopoietic components of the thymus and is arranged into 2 regions, the cortex and medulla
    • Stromal components comprise the thymic structure and provide a matrix for thymocyte development
    • Keratin positive cells: cortical thymic epithelial cells (cTECs) and medullary TECs (mTECs)
    • Keratin negative cells: fibroblasts, nonfibroblastic mesenchymal cells, endothelial cells of the vasculature and connective tissue that forms the capsule and septa
    • Other stromal cells: Hassall corpuscles, dendritic cells, macrophages and CD45+ hematopoietic cells
    • Mature thymus will show many clusters of adipose cells where the process of involution has replaced the thymic parenchyma with fat (Gerontology 2005;51:14)
  • Hassall corpuscles are unique to the thymus and are involved in thymocyte maturation, clearing of apoptotic cells and lymphopoiesis (Age (Dordr) 2014;36:313)
    • Concentrically arranged type VI epithelial reticular cells
    • Reside in the medulla, not in cortex
    • Well developed in mature thymus
  • Blood thymus barrier is composed of several layers of cells; these include vascular endothelial cells, pericytes, squamous TECs connected by desmosomes and occluding junctions, macrophages, cTECs, mTECs and stellate TECs (StatPearls: Anatomy, Head and Neck - Blood Thymus Barrier [Accessed 26 February 2024])
  • Reference: Z Gerontol Geriatr 2000;33:341
Microscopic (histologic) images

Contributed by Nicole Stringham, Ph.D. (source: University of Michigan virtual slide box and Duke University virtual slide collection)
Neonatal thymus, lobule

Neonatal thymus, lobule

Neonatal thymus, cortex / medulla

Neonatal thymus, cortex / medulla

Neonatal thymus, Hassall corpuscle

Neonatal thymus, Hassall corpuscle

Adult thymus, involuted

Adult thymus, involuted

Adult thymus, medulla

Adult thymus, medulla

Adult thymus, Hassall corpuscle

Adult thymus, Hassall corpuscle

Virtual slides

Images hosted on other servers:
Thymus, adult

Thymus, adult

Thymus, young

Thymus, young

Cytology description
  • Hallmark cytology of thymus includes 2 cell types, lymphocytes and epithelial cells (Cancers (Basel) 2022;14:2013)
  • Diagnosis of thymic hyperplasia is challenging on cytology due to the difficulty of identifying Hassall corpuscles and because smear cellularity can affect the size and density of epithelial clusters
  • Thymic hyperplasia: epithelial cell clusters are more sparse and smaller than those of thymoma and can mimic lymphoid tangles in lymph node aspirates
    • Presence of Hassall corpuscles favors hyperplasia over thymoma (although they may be seen in B1 thymoma)
Cytology images

Images hosted on other servers:

Thymic hyperplasia

Positive stains
Negative stains
Videos

Mediastinum

Histology of the thymus

Board review style question #1
Which anatomical compartment contains the aortic arch, brachiocephalic trunk, left common carotid and left subclavian arteries?

  1. Anterior mediastinum
  2. Middle mediastinum
  3. Posterior mediastinum
  4. Superior mediastinum
Board review style answer #1
D. Superior mediastinum contains aortic arch, brachiocephalic trunk, left common carotid and left subclavian arteries as well as the esophagus, trachea, thymus, phrenic and vagus nerves. Answer A is incorrect because anterior mediastinum notably contains the internal thoracic arteries and transversus thoracis muscles. Answer B is incorrect because middle mediastinum contains the heart and ascending aorta but not the arch. Answer C is incorrect because posterior mediastinum contains the descending aorta and other structures that pass from the thorax into the abdomen.

Comment Here

Reference: Mediastinum - Anatomy & thymus histology
Board review style question #2

The micro image shown above is in need of evaluation. Features that are present, both in the image and in adjacent areas, include a connective tissue capsule, cortex and medulla, efferent lymphatics (only) and concentrically arranged epithelial reticular cells within the medulla. What is the anatomical origin of this sample?

  1. Lymph node
  2. Spleen
  3. Thymus
  4. Thyroid
  5. Tonsil
Board review style answer #2
C. Thymus. The thymus is the only listed option that has numerous lymphocytes (shown in the image) in addition to all of the other listed features. Additionally, the concentrically arranged epithelial reticular cells (i.e., Hassall corpuscles) are unique to the medulla of the thymus. Answer A is incorrect because although lymph nodes have many of these features, including a cortex / medulla arrangement, they do not have Hassall corpuscles. Answer B is incorrect because spleen is histologically arranged as red pulp and white pulp, with nodules throughout the organ, not as a cortex and medulla. Answer E is incorrect because tonsils contain aggregates of lymphocytes but do not have many of the features listed. Answer D is incorrect because thyroid histomorphology would show simple columnar epithelium arranged in spheres around colloid rather than a parenchyma of lymphocytes.

Comment Here

Reference: Mediastinum - Anatomy & thymus histology

Bronchogenic cyst
Definition / general
  • Benign congenital malformation of tracheobronchial tree
Essential features
  • Benign congenital malformation arising from abnormal budding of primitive foregut during gestation
  • Occurs in mediastinum, lung, head and neck, skin, abdomen, retroperitoneum
  • Unilocular cyst lined by respiratory type epithelium
  • Cyst wall recapitulates bronchial wall with variable amounts of seromucinous glands, cartilage and smooth muscle
  • Excellent prognosis with complete surgical excision
Terminology
  • Bronchial cyst
  • Noncommunicating bronchopulmonary foregut malformation
ICD coding
  • ICD-10: J98.4 - other disorders of lung
  • ICD-11: CB40.Y - other specified diseases of the respiratory system
Epidemiology
Sites
  • Depends on stage of development when malformation occurs
  • Most common in thorax: mediastinum (early in gestation), lung (later in gestation)
  • Ectopic locations: head and neck (usually midline; less commonly lateral or subcutaneous), abdomen, retroperitoneum (Acta Otorhinolaryngol Ital 2009;29:36, Am Surg 2005;71:1034)
Pathophysiology
  • Arises from abnormal budding of primitive ventral foregut
  • Usually occurs between 26 - 40 days of gestation
Etiology
  • No known causes or risk factors
Clinical features
  • Most present with symptomatic disease
    • Approximately 50% present with pain (Lung 2008;186:55, Am Surg 2005;71:1034)
    • Respiratory symptoms (cough, dyspnea, hemoptysis) (Lung 2008;186:55)
    • Respiratory distress may result from tracheal compression
    • Gastrointestinal symptoms (dysphagia, nausea, vomiting)
    • Occasionally fever, other constitutional symptoms or infection
  • Asymptomatic presentation uncommon
    • True prevalence of asymptomatic disease unknown
Diagnosis
Radiology description
Radiology images

Images hosted on other servers:

Fluid filled mediastinal mass

Mediastinal mass with high signal intensity on T1 and T2

Mediastinal bronchogenic cyst

Well circumscribed
intrapulmonary
bronchogenic cyst

Esophageal bronchogenic cyst

Prognostic factors
Case reports
Treatment
  • Complete surgical excision recommended for symptomatic patients
  • Aspiration or ablation for nonresectable lesions / high risk patients (J Bronchology Interv Pulmonol 2015;22:195)
  • Observation acceptable for asymptomatic patients
Clinical images

Images hosted on other servers:

Submucosal esophageal bronchogenic cyst

Polypoid endobronchial bronchogenic cyst

Gross description
  • Unilocular thin walled cyst with smooth surface
  • Filled with serous or mucinous fluid
  • May be turbid or hemorrhagic if infected or previously aspirated / biopsied
  • Reference: Semin Pediatr Surg 2003;12:17
Gross images

Images hosted on other servers:

Unilocular cyst with mucoid material

Microscopic (histologic) description
  • Cyst lined by respiratory type epithelium
    • Ciliated pseudostratified columnar cells
    • Variable numbers of goblet cells
    • Filled with serous or mucinous material
  • Cyst wall components (variable)
    • Seromucinous glands
    • Hyaline cartilage
    • Smooth muscle
  • Changes associated with infection or prior procedure
    • Acute and chronic inflammation with epithelial denudation
    • Hemorrhage with hemosiderin laden macrophages
    • Squamous metaplasia
    • Cholesterol clefts
    • Fibrosis
  • Reference: Semin Pediatr Surg 2003;12:17
Microscopic (histologic) images

Contributed by Laurence M. Briski, M.D.

Unilocular cyst

Recapitulation of bronchial structure

Cyst wall components

Foci of smooth muscle

Ciliated respiratory epithelium

Cytology description
  • Ciliated columnar (respiratory type) epithelial cells
  • Macrophages (and other cystic contents)
  • Acute and chronic inflammatory cells
  • Serous or mucinous material
  • Rarely cartilage or smooth muscle spindle cells
  • Reference: J Bronchology Interv Pulmonol 2015;22:195
Cytology images

Images hosted on other servers:

Cystic contents (Diff-Quik stain)

Positive stains
Negative stains
Sample pathology report
  • Soft tissue, thorax, excision:
    • Bronchogenic cyst
Differential diagnosis
  • Cystic teratoma:
    • Usually has a solid component
    • Lined by variable mixtures of gastrointestinal, squamous and respiratory epithelium
    • Contains tissues derived from all 3 germ cell layers
    • Some cases contain immature tissues, nuclear atypia or tumor necrosis
  • Thyroglossal duct cyst:
    • Lined by respiratory, cuboidal or squamous epithelium
    • Contains thyroid tissue (follicles) in cyst wall
    • Lacks cartilage and smooth muscle
  • Esophageal duplication cyst:
    • Lined by stratified squamous or gastrointestinal epithelium
    • Contains double smooth muscle layer
    • Attached to esophageal wall
  • Branchial cleft cyst:
    • Typically lined by stratified squamous epithelium
    • Contains lymphoid aggregates with reactive germinal centers
    • Often filled with keratinaceous debris
    • Lacks cartilage and smooth muscle
  • Dermoid cyst:
    • Lined by stratified squamous epithelium
    • Contains hair and skin appendages
    • Filled with keratinaceous or sebaceous material
    • Lacks cartilage and smooth muscle
  • Abscess:
    • No true lining; may contain foci of squamous metaplasia
    • Filled with suppurative inflammation
    • Lacks cartilage and smooth muscle
Board review style question #1

A 1 month old girl presents with acute respiratory distress and failure to thrive. Imaging studies reveal a 3.5 cm cystic mediastinal mass compressing her trachea. The lesion is excised and shown above.

Which of the following statements accurately describes this lesion?

  1. Congenital cyst arising from abnormal budding of the primitive ventral foregut
  2. Congenital cyst derived from remnants of the second branchial apparatus
  3. Congenital cyst derived from the thyroglossal duct
  4. Mature germ cell tumor derived from all 3 germ cell layers
Board review style answer #1
A. Congenital cyst arising from abnormal budding of the primitive ventral foregut. The lesion shown is a bronchogenic cyst. Teratomas are mature germ cell tumors derived from all 3 germ cell layers (answer D); branchial cleft cysts are derived from remnants of the second branchial apparatus (answer B); thyroglossal duct cysts are derived from the thyroglossal duct (answer C).

Comment Here

Reference: Bronchogenic cyst
Board review style question #2
Each of the following immunohistochemical markers may be positive in cystic teratomas of the mediastinum. Which is most reliably negative in bronchogenic cysts and therefore useful in the distinction between these 2 entities?

  1. CDX2
  2. Cytokeratin 7
  3. Cytokeratin 20
  4. TTF1
Board review style answer #2
A. CDX2. Based on one series of 22 bronchogenic cysts, CDX2 was negative in all 22 cases. The other listed immunohistochemical stains had varying levels of positivity in this cohort (from positivity with CK7 in 100% of cases to positivity with TTF1 in 18% of cases) (Am J Clin Pathol 2008;130:265).

Comment Here

Reference: Bronchogenic cyst

Carcinoid tumor
Definition / general
  • Similar to carcinoid tumors elsewhere
Terminology
  • Also called well differentiated neuroendocrine carcinoma
Sites
  • Usually of thymic origin in anterior mediastinum, occasionally in middle or posterior mediastinum
Diagrams / tables

Images hosted on other servers:

Features of 4 patients
(tables 1 and 3)

Clinical features
  • Mean age 48 years old, 80% men
  • 20% occur in MEN1 or MEN2 patients
  • One - third have paraneoplastic Cushing syndrome, syndrome of inappropriate antidiuretic hormone, Eaton-Lambert syndrome or rarely PTH production (Ann Thorac Surg 2002;73:675)
  • Coexisting carcinoid syndrome is very unusual
  • Behavior similar to atypical carcinoid at other sites; must be considered to have metastatic potential with metastasis to mediastinal lymph nodes, bone, liver, skin
  • May recur after up to 10 years
Prognostic factors
  • Poor prognosis (Korean J Pathol 2012;46:221)
  • Poor prognostic factors: ectopic ACTH production or association with MEN syndromes
  • 5 year survival: 70% without endocrinopathy versus 35% with endocrinopathy
Case reports
Treatment
  • Aggressive, presents at advanced stage; complete surgical resection with postoperative radiotherapy to tumor bed is recommended (Interact Cardiovasc Thorac Surg 2010;11:732)
  • Radiation therapy and chemotherapy are ineffective
Gross description
  • Well circumscribed but unencapsulated, firm, gray-pink, fleshy, gritty on cut section, hemorrhage, necrosis, no internal fibrous septa
Microscopic (histologic) description
  • Organoid pattern with islands, ribbons, festoons, trabeculae, rosettes of small round cells with minimal cytoplasm, salt and pepper chromatin, no / rare mitotic activity
  • Cellular nests may become detached from septa during processing and contain foci of central geographic necrosis with dystrophic calcification
  • Marked vascularization, frequent angiolymphatic invasion, may have amyloid type stroma, sclerotic (desmoplastic type) stroma, melanin pigment, mucin
  • May coexist with sarcomatoid carcinoma, thymoma, thymic cyst
  • No lymphocytes, no perivascular spaces
Microscopic (histologic) images

Case #278

Atypical carcinoid

Atypical carcinoid -
AE1 / AE3

Atypical carcinoid - synaptophysin



Images hosted on other servers:

Figures a - d

Atypical carcinoid

Various stains

Positive stains
Electron microscopy description
  • Numerous dense core neurosecretory granules, perinuclear whorls of intermediate filaments, intercellular junctions
  • No complex nucleoli, no premelanosomes, no branching surface microvilli, no complex desmosomes, no tonofilaments
Differential diagnosis
Spindle cell carcinoid of thymus
Clinical features
  • Very rare
  • May cause death
  • Associated with MEN1, syndrome of inappropriate antidiuretic hormone secretion
  • Aggressive clinical behavior (Mod Pathol 1999;12:587)

Case reports
Gross description
  • Tan-brown, well circumscribed, encapsulated, 2 - 15 cm

Microscopic (histologic) description
  • Fascicles of plump spindle cells separated by thin fibrovascular septa
  • Amphophilic and granular cytoplasm, finely dispersed chromatin
  • Frequent mitotic figures, frequent focal necrosis
  • No / vague organoid pattern, no prominent nucleoli

Positive stains
Electron microscopy description
  • Neurosecretory granules

Enteric (esophageal) duplication cyst
Definition / general
  • Children or teens, with cysts usually in posterior mediastinum, due to developmental defect from fusion of tracheoesophageal septum
  • Gastric cysts may produce acid and rupture or hemorrhage
Clinical features
  • Associated with vertebral anomalies (hemivertebrae, spina bifida)
  • Leaky cysts are associated with pleural effusion or pulmonary consolidation
  • Paraesophageal cysts are associated with dysphagia or subnormal weight gain
  • Gastroesophageal cysts are associated with cough, vomiting, fever, pneumonia, empyema
Case reports
Gross description
  • 2 - 10 cm, rounded or irregular, with fibromuscular wall of variable thickness
  • Usually unilocular but may be multiloculated
  • Smooth inner lining, often mucoid contents
Microscopic (histologic) description
  • Squamous, simple columnar, pseudostratified columnar or mixed epithelial lining, usually with some gastric glandular mucosa, overlying a double layer of smooth muscle
  • No cholesterol granulomas, no cartilage

Germ cell tumors
Definition / general
  • Account for 1 - 15% of all mediastinal neoplasms in adults and 11 - 20% in children (J Thorac Oncol 2020;15:568, Ann Surg Oncol 1994;1:121)
  • Classified as seminomas and nonseminomatous germ cell tumors (yolk sac tumor, embryonal carcinoma, choriocarcinoma, teratoma and mixed germ cell tumor)
  • A seminoma with any component of a nonseminomatous tumor is considered a nonseminomatous germ cell tumor
Essential features
  • Mature teratomas and yolk sac tumors are almost exclusively the tumor types in very young patients
  • Seminomas are almost always diagnosed in patients ≥ 10 years old
  • Mixed mediastinal germ cell tumors more common in adults
  • Majority occur in anterior mediastinum
  • On CT, presence of fat is suggestive of their presence; however, fat fluid level or bone is pathognomonic of teratoma
  • Immunoprofile of primary mediastinal germ cell tumors is similar to their gonadal counterparts
Terminology
  • Seminoma
  • Nonseminomatous germ cell tumors (NSGCT) include yolk sac tumor, embryonal carcinoma, choriocarcinoma, teratoma and mixed germ cell tumor
ICD coding
  • ICD-O:
    • 9061/3 - seminoma
    • 9070/3 - embryonal carcinoma
    • 9071/3 - yolk sac tumor
    • 9100/3 - choriocarcinoma
    • 9080/0 - mature teratoma
    • 9080/1 - immature teratoma of the thymus
    • 9085/3 - mixed germ cell tumor
    • 9084/3 - teratoma with somatic type malignancies
    • 9086/3 - germ cell tumor with associated hematological malignancy
Epidemiology
  • Median age of mediastinal seminoma is 33 years, while NSGCT is 28 years (J Clin Oncol 2002;20:1864)
  • Mature teratomas and yolk sac tumors are almost exclusively the tumor types in very young patients
  • Seminomas are almost always diagnosed in patients aged ≥ 10 years (Pediatr Blood Cancer 2004;42:169)
  • Mixed mediastinal germ cell tumors more common in adults
  • M > F (Front Oncol 2020;10:1137)
  • Benign teratomas have equal gender distribution
Sites
Pathophysiology
  • Thought to arise from aberrant migration of primitive germ cells during embryonic development or from stem cells in thymus (Int J Androl 2007;30:256)
Etiology
Diagrams / tables

Gross, micro, IHC and serum markers of germ cell tumors
Tumor type Gross features Microscopic features Serum markers Pankeratin SALL4 OCT3/4 CD30 Glypican 3 Beta hCG CD117
Teratoma Solid combined with cystic, Rokitansky protuberans contains shiny cartilage, hair and teeth, etc. Derivatives of all 3 germinal layers Beta hCG: normal
LDH: normal
AFP: normal
Positive in epithelial elements Negative in mature components Negative in mature components Negative Negative Negative Negative
Embryonal carcinoma Gray-white with areas of hemorrhage and necrosis Sheets and aggregates of highly pleomorphic cells Beta hCG: normal
LDH: raised
AFP: raised
Positive Positive Positive Positive Negative Negative Negative
Yolk sac tumor Gray-white with variable hemorrhage Numerous growth patterns, Schiller-Duval bodies, eosinophlic globules Beta hCG: normal
LDH: normal
AFP: raised
Positive Positive Negative Negative Positive Positive in syncytio-trophoblasts Negative
Chorio-carcinoma Markedly hemorrhagic Trimorphic population of trophoblasts with extensive hemorrhage Beta hCG: raised
LDH: normal
AFP: normal
Positive Negative Negative Negative Negative Positive in syncytio-trophoblasts Negative
Seminoma Gray-white, shiny cut surface Islands of pleomorphic cells with prominent nucleoli, separated by thin fibrous septa having lymphocytes Beta hCG: raised
LDH: raised
AFP: normal
Negative Positive Positive Negative Negative Negative Positive
Clinical features
  • Chest pain, cough, dyspnea, superior vena cava syndrome, weight loss, cervical mass, chest wall mass
  • Lung involvement, liver and bone metastasis (Front Oncol 2020;10:1137)
Diagnosis
  • Clinical symptoms and elevated serum markers
  • Radiological examination indicating a mass most commonly in anterior mediastinum
Laboratory
  • Lactate dehydrogenase (LDH)
  • Alpha fetoprotein (AFP)
  • Beta human chorionic gonoadotropin (hCG)
Choriocarcinoma
Definition / general
  • Highly malignant trophoblastic neoplasm composed of syncytiotrophoblasts, cytotrophoblasts and variable number of intermediate trophoblasts

Essential features

ICD coding
  • ICD-O: 9100/3 - choriocarcinoma
  • ICD-11: 2C28.0 & XH8PK7 - malignant germ cell neoplasms of heart, mediastinum or nonmesothelioma of pleura & choriocarcinoma, NOS

Epidemiology
Sites

Etiology
Clinical features
Diagnosis
  • Radiological imaging with serum markers and biopsy or excision

Laboratory
  • Elevated serum beta hCG levels

Radiology description
Radiology images

Images hosted on other servers:

Mediastinal mass & metastatic lung nodules

Mass in posterior mediastinum



Prognostic factors
Case reports
  • 25 year old man presented with cutaneous metastasis of primary mediastinal choriocarcinoma (J Cutan Pathol 2021;48:81)
  • 26 year old man with primary mediastinal choriocarcinoma with diffuse metastasis to both lungs and multiple brain metastases (Medicine (Baltimore) 2019;98:e16411)
  • 71 year old man on goserelin treatment for metastatic prostatic adenocarcinoma developed mediastinal choriocarcinoma with lung and vertebral metastases (Case Rep Pathol 2019;2019:2734815)

Treatment
  • Chemotherapy, radiotherapy and surgery
  • Treatment with cisplatin based therapy may improve the outcome

Gross description
  • Large, soft, friable, extensively hemorrhagic and with foci of necrosis

Microscopic (histologic) description
  • Intermingled syncytiotrophoblasts and cytotrophoblasts in a plexiform pattern or in disordered sheets
  • Syncytiotrophoblasts, large multinucleated cells with numerous, pleomorphic, dark staining nuclei, variably distinct nucleoli and abundant densely eosinophilic or amphophilic cytoplasm
  • Cytotrophoblasts, uniform, polygonal cells with round nuclei, prominent nucleoli and clear or eosinophilic cytoplasm
  • Dilated vascular sinusoids
  • Extensive areas of hemorrhage and necrosis
  • Atypical mitoses and cellular atypia are common (J Cutan Pathol 2021;48:81, Mediastinum 2019;3:30)
  • Mostly choriocarcinoma occurs as a component of mixed germ cell tumors (Arch Pathol Lab Med 2019;143:65)

Microscopic (histologic) images

Contributed by Sarosh Moeen, M.B.B.S.
Intermediate sized cells

Intermediate sized cells

Pankeratin (CK AE1 / AE3)

Pankeratin (CK AE1 / AE3)



Cytology description
  • Abundant necrosis and hemorrhage with clusters or isolated multinucleated syncytiotrophoblasts

Cytology images

Images hosted on other servers:

Transbronchial aspirate: syncytiotrophoblast



Positive stains
Negative stains
Sample pathology report
    • Anterior mediastinal mass, core biopsy:
      • Germ cell tumor with morphological and immunohistochemical profile favoring choriocarcinoma (see comment)
      • Comment: A possibility of mixed germ cell tumor in the vicinity cannot be excluded.

    Differential diagnosis
    • Sarcomatous carcinoma with giant cells:
      • Shows small to medium sized tumor cells with larger giant cells; this is in contrast with choriocarcinomas that show 2 cell populations of cytotrophoblasts and syncytiotrophoblasts
Embryonal carcinoma
Definition / general
  • Malignant nonseminomatous germ cell tumor characterized by embryonal type cells

ICD coding
  • ICD-O: 9070/3 - embryonal carcinoma
  • ICD-11: 2C28.0 & XH8MB9 - malignant germ cell neoplasms of heart, mediastinum or nonmesothelioma of pleura & embryonal carcinoma, NOS

Epidemiology
  • Accounts for 2 - 8% of primary mediastinal germ cell tumors (Cancer 2003;97:367)
  • Predominantly in young males

Sites
  • Occurs in anterior mediastinum

Etiology
  • Same as other nonseminomatous germ cell tumors

Clinical features
  • Chest pain, dyspnea, cough and superior vena cava syndrome

Diagnosis
  • Mediastinal mass on radiology and subsequent biopsy

Laboratory
  • Elevated LDH and AFP levels

Radiology description
  • Large, lobulated, heterogeneous mass with hemorrhage, necrosis and calcifications on CT scan
  • On MRI, cystic or necrotic areas appear hyperintense on T2 weighted images
  • FDG PET useful in evaluation of recurrent disease or residual viable tumor after chemotherapy (AJR Am J Roentgenol 2010;195:W274)

Radiology images

Images hosted on other servers:

Expansion of tumor; heterogeneously enhanced



Prognostic factors
Case reports
  • 12 year old boy with large soft tissue mass in mediastinum and left thoracic cavity, diagnosed as malignant mixed germ cell tumor with yolk sac tumor, immature teratoma and embryonal carcinoma components (Open Med (Wars) 2021;16:892)
  • 23 year old man with anterior mediastinal mass and right pleural effusion developed pericardial effusion and cardiac tamponade (AJR Am J Roentgenol 1998;170:722)
  • 25 year old man with an unresectable embryonal carcinoma of the anterior mediastinum that developed a bronchial fistula after systemic chemotherapy (Case Rep Radiol 2020;2020:7650206)
  • 29 year old man with primary mediastinal embryonal carcinoma presented with pulmonary artery stenosis (Pneumonol Alergol Pol 2015;83:151)

Treatment
Gross description
  • Infiltrating and large with size up to 22 cm
  • Hemorrhagic and necrotic cut surface

Microscopic (histologic) description
  • Large and polygonal cells sometimes columnar with indistinct cell borders
  • Eosinophilic, amphophilic, basophilic or clear cytoplasm
  • Nuclei have large single or multiple nucleoli (Histopathology 2022;80:381)
  • Numerous mitoses, often atypical
  • Solid, tubular / glandular or papillary growth patterns
  • Tumor necrosis is common (Mediastinum 2019;3:30)

Microscopic (histologic) images

Contributed by Sarosh Moeen, M.B.B.S.
Islands of tumor cells

Islands of tumor cells

Pleomorphic cells

Pleomorphic cells

Pankeratin CK AE1 / AE3

Pankeratin CK AE1 / AE3

SALL4

SALL4


OCT3/4

OCT3/4

Glypican 3

Glypican 3

CD30

CD30



Cytology description
  • Syncytial cohesive clusters of pleomorphic cells with frequent mitoses
  • Necrosis and hemorrhage in many cases (Cancer 2008;114:504)

Cytology images

Images hosted on other servers:

Pleomorphic tumor cells; marked nuclear pleomorphism



Positive stains
Negative stains
Sample pathology report
  • Anterior mediastinal mass, core biopsy:
    • Germ cell tumor with morphological and immunohistochemical profile favoring embryonal carcinoma (see comment)
    • Comment: A possibility of mixed germ cell tumor in the vicinity cannot be excluded.

Differential diagnosis
Mixed germ cell tumors of the mediastinum
Definition / general
  • Neoplasms composed of 2 or more types of germ cells

Essential features
  • Presence of more than 1 type of germ cell tumor

Terminology
  • Malignant teratoma (not recommended)

ICD coding
  • ICD-O: 9085/3 - mixed germ cell tumor
  • ICD-11: 2C28.0 & XH2PS1 - malignant germ cell neoplasms of heart, mediastinum or nonmesothelioma of pleura & mixed germ cell tumor

Epidemiology
  • In adults, M > F and common tumor components are teratoma and embryonal carcinoma
  • In children, yolk sac tumor and teratoma (mature or immature) are the most common types

Sites
  • Occur in anterior mediastinum

Etiology
  • Similar to other types

Clinical features
  • Mediastinal mass and elevated serum markers

Diagnosis
  • Elevated serum markers
  • Mediastinal mass on imaging
  • Biopsy showing 2 or more types of germ cell components

Laboratory

Radiology description

Radiology images

Images hosted on other servers:
CT scan of chest with anterior mediastinal mass


CT scan of chest with anterior mediastinal mass



Case reports
  • 12 year old boy with mixed germ cell tumor of the mediastinum diagnosed on core biopsy (Open Med (Wars) 2021;16:892)
  • 16 year old boy with huge mediastinal mass diagnosed as mixed germ cell tumor with possible combination of embryonal carcinoma, yolk sac and teratoma (Case Rep Surg 2016;2016:7615029)
  • 26 year old man with mixed germ cell tumor comprising of immature teratoma and seminoma diagnosed due to massive organ displacing tumor in right chest on chest Xray (Pathol Res Pract 2009;205:572)
  • 29 year old man with disease progression despite systemic chemotherapy, diagnosed as mixed germ cell tumor on en bloc resection (J Surg Case Rep 2021;2021:rjab416)

Treatment
  • Resected after chemotherapy

Gross description
  • Heterogeneous cut surface
  • Solid, fleshy tumor with areas of hemorrhage and necrosis
  • Cystic spaces indicate presence of a teratomatous component

Gross images

Images hosted on other servers:
4.6 kg mediastinal mixed germ cell tumor, mostly teratoma

4.6 kg mediastinal mixed GCT, mostly teratoma



Microscopic (histologic) description
  • Morphology identical to pure germ cell component
  • Please refer to the sections above for microscopic description of individual components

Cytology description
  • Please refer to the sections above for cytological description of individual components

Positive stains
  • Please refer to the sections above for individual components

Negative stains
  • Please refer to the sections above for individual components

Sample pathology report
  • Anterior mediastinal mass, biopsy:
    • Malignant mixed germ cell tumor with seminoma (~50%), yolk sac tumor (~30%) and embryonal carcinoma (~20%) components (see comment)
    • Comment: The findings show majority of tumor comprised of sheets of polygonal cells with vesicular nuclei and prominent nucleoli. Intervening fibrous septa show lymphocytic infiltration. This component stains positive for immunohistochemical stains SALL4 and OCT3/4 while staining negative for CD30 and glypican 3. Another admixed component shows reticular pattern and Schiller-Duval bodies of cuboidal cells which stain positive for SALL4 and glypican 3 while staining negative for OCT3/4 and CD30. A small component comprised of large polygonal cells with pleomorphic nuclei, prominent nucleoli, numerous mitoses and necrosis is seen. This component stains positive for SALL4 and OCT3/4 and CD30 while staining negative for glypican 3.
    • See synoptic reporting for resection specimen.
Seminoma
Definition / general
  • Malignant germ cell tumor composed of seminomatous germ cells
  • A seminoma with any component of a nonseminomatous tumor is considered a nonseminomatous germ cell tumor (NSGCT)

Essential features

Terminology
  • Germinoma

ICD coding
  • ICD-O: 9061/3 - seminoma
  • ICD-11: 2C28.0 & XH9FM4 - malignant germ cell neoplasms of heart or mediastinum, or nonmesothelioma of pleura & seminoma, NOS

Epidemiology
Sites

Etiology
Clinical features
  • Most patients are symptomatic
  • Dyspnea, chest pain, superior vena cava syndrome, cough with hemoptysis, hoarseness

Diagnosis
Laboratory
Radiology description
Radiology images

Images hosted on other servers:

Mass (M) in prevascular mediastinum



Prognostic factors
  • Prognosis is better than nonseminomatous malignant germ cell tumors
  • Good prognosis if nonpulmonary visceral metastasis is absent (J Clin Oncol 2002;20:1864)

Case reports
Treatment
  • Cisplatin based combination chemotherapy and radiotherapy
  • Surgical excision is reserved for cases not responding to chemotherapy and radiotherapy (Oncol Res Treat 2019;42:95)

Gross description
  • Smooth and glistening or lobulated outer surface
  • Soft, creamy to light tan nodular cut surfaces (Kaohsiung J Med Sci 2005;21:395)
  • May show both solid and cystic areas with necrotic material

Gross images

Images hosted on other servers:

Cystic seminoma with necrotic tissue



Microscopic (histologic) description
  • Round to polygonal, uniform epithelioid cells with round to oval nuclei, vesicular chromatin and large nucleoli
  • Abundant, glycogen rich, clear to lightly eosinophilic cytoplasm and distinct cell membranes
  • Tumor cells grow in confluent sheets, multinodular clusters, cords or irregular lobules
  • Lymphocytic infiltration of fibrous septa / stroma is almost always seen
  • Other findings including nonnecrotizing granulomas, cellular pleomorphism, necrosis, intercellular edema, syncytiotrophoblasts (Kaohsiung J Med Sci 2005;21:395, Int J Clin Exp Pathol 2019;12:3082)

Microscopic (histologic) images

Contributed by Sarosh Moeen, M.B.B.S.
Uniform appearing cells

Uniform appearing cells



Cytology description
  • Discohesive cells, round or polygonal nuclei, 1 or more prominent nucleoli, pale cytoplasm
  • Lymphocytes, plasma cells and characteristic tigroid stripes
  • Necrosis and giant cells may be present (Cancer 2008;114:504)

Cytology images

Images hosted on other servers:

Clusters of cells; tigroid background

Cytologic smear; cell block



Positive stains
Negative stains
Sample pathology report
  • Anterior mediastinal mass, core biopsy:
    • Germ cell tumor with morphological and immunohistochemical profile favoring seminoma (see comment)
    • Comment: A possibility of mixed germ cell tumor in the vicinity cannot be excluded.

Differential diagnosis
Teratoma
Definition / general
  • Neoplasm of pluripotent cell origin that forms differentiated somatic type tissues, which may be exclusively mature (adult type), exclusively immature (embryonal or fetal) or a combination of both

Essential features
  • Tumor composed entirely of differentiated somatic type tissue, may be mature, immature or combined

ICD coding
  • ICD-O:
    • 9080/0 - mature teratoma
    • 9080/1 - immature teratoma of the thymus
  • ICD-11:
    • 2C28.0 & XH3GV5 - malignant germ cell neoplasms of heart, mediastinum or nonmesothelioma of pleura & teratoma, benign
    • 2C28.0 & XH7YZ9 - malignant germ cell neoplasms of heart, mediastinum or nonmesothelioma of pleura & teratoma, malignant, NOS

Epidemiology
Sites
  • Occurs in anterior mediastinum and rarely in posterior mediastinum

Etiology
  • Limited data about molecular biology and genetic features of primary mediastinal teratomas

Clinical features
  • Chest pain, shortness of breath, wheezing and cough due to compression of adjacent structures
  • Superior vena cava syndrome, hemoptysis, dysphagia, nausea and hoarseness are less common

Diagnosis
  • Incidental mass on radiological imaging in asymptomatic patients with serum tumor markers (AFP and beta hCG) not elevated
  • More symptomatic in children
  • Characteristic radiological findings of soft tissue, fat, fluid and calcifications in variable proportions
  • Biopsy findings showing differentiated somatic type tissue, may be mature, immature or combined

Laboratory
  • Serum tumor markers AFP and beta hCG are not elevated

Radiology description
Radiology images

Images hosted on other servers:

Huge posterior mediastinal mass

Anterior mediastinal mass



Prognostic factors
  • Mature teratomas are benign
  • Immature teratomas have malignant potential and malignant behavior correlates with amount of immature tissue
  • Somatic type malignancy may occur in mature teratomas, which determines the prognosis (Int J Surg Case Rep 2021;80:105680)

Case reports
Treatment
  • Complete surgical excision in cases of benign teratoma

Clinical images

Images hosted on other servers:

En bloc tumor resection



Gross description
  • Encapsulated masses with variegated cut surface and unilocular or multilocular cysts
  • Cyst contents may include clear fluid, mucoid material, sebaceous and keratinaceous debris, hair, fat, cartilage and rarely teeth or bone
  • Immature teratomas have soft to fleshy consistency or are extensively fibrous or cartilaginous
  • Hemorrhage and necrosis common

Gross images

Images hosted on other servers:

Excised tumor with rib



Microscopic (histologic) description
  • Haphazard distribution of mature somatic tissue is hallmark
  • Skin and cutaneous appendages are common, often lining cysts
  • Bronchial mucosa, glands, gastrointestinal mucosa, nerves and mature brain tissue
  • Smooth muscle, adipose tissue, mature pancreatic tissue
  • Immature tissue corresponds to embryonal or fetal tissues either exclusively or in addition to mature tissue
  • Most common immature tissue is neuroectodermal tissue with neuroepithelial cells forming tubules and rosettes (Mediastinum 2019;3:30)
  • Fetal lung, mesenchyme, primitive cartilage, bone, rhabdomyoblasts may be seen

Microscopic (histologic) images

Contributed by Sarosh Moeen, M.B.B.S.
Skin and choroid plexus

Skin and choroid plexus

Mucinous epithelium

Mucinous epithelium

Respiratory and squamous epithelium

Respiratory and squamous epithelium

Glial tissue

Glial tissue

Mature bone

Mature bone


Cartilage lobule

Cartilage lobule

Adipose tissue and acini

Adipose tissue and acini

Odontogenic tissue

Odontogenic tissue

Pancreatic acini

Pancreatic acini



Cytology description
  • Anucleate squamous cells and macrophages in a cystic background
  • Presence of ciliated bronchial epithelium, smooth muscle and cartilage may be mistaken for contamination
  • Presence of pancreatic acini or intestinal type mucosa may be suggestive of a teratomatous lesion
  • Careful correlation with imaging and serological studies to exclude mixed germ cell tumor (Cancer 2008;114:504)

Cytology images

Images hosted on other servers:

Mature and immature teratoma



Positive stains
  • Not supportive in mature teratoma

Negative stains
Videos

Pathology mini tutorial for teratoma



Sample pathology report
  • Anterior mediastinal mass, core biopsy:
    • Germ cell tumor with features favoring mature teratoma (see comment)
    • Comment: The microscopic features show skin adnexal structures, mature cartilage and a tiny focus of mature glial tissue.
    • A possibility of immature component or mixed germ cell tumor in the vicinity cannot be excluded.

Differential diagnosis
Yolk sac tumor
Definition / general
  • Malignant nonseminomatous germ cell tumor characterized by numerous patterns that recapitulate the yolk sac, allantois and extraembryonic mesenchyme

Terminology
  • Endodermal sinus tumor (not recommended)

ICD coding
  • ICD-O: 9071/3 - yolk sac tumor
  • ICD-11: 2C28.0 & XH09W7 - malignant germ cell neoplasms of heart, mediastinum or nonmesothelioma of pleura & yolk sac tumor

Epidemiology
  • Restricted to males after puberty and strong female preponderance in children
  • Peak incidence at third decade of life in adults
  • Pure yolk sac tumor 2 - 12% of mediastinal germ cell tumor in adults
  • Yolk sac tumor is virtually the only malignant germ cell tumor of the mediastinum in children up to 5 years of age (Turk Pediatri Ars 2019;54:185, Ann Diagn Pathol 2021;53:151763)

Sites
  • Most commonly anterior mediastinum

Etiology
  • Klinefelter syndrome is a risk factor after puberty (Int J Pediatr Endocrinol 2020;2020:18)
  • Genetic losses at 1p, 4q and 6q and gains at 1q, 3, 20q and 20 reported in mediastinal yolk sac tumors in patients < 8 years of age
  • After 8 years of age, isochromosome 12p, gain of chromosome 21 and X and loss of chromosome 13

Clinical features
  • Chest pain, dyspnea, chills, fever and superior vena cava syndrome

Diagnosis
  • FDG PET may detect tiny occult primary tumor or recurrences

Laboratory
Radiology description
  • Large solid - cystic mass with intratumoral hemorrhage, capsular tear, marked heterogeneous enhancement and enlarged intratumoral vessels on CT scan (Acta Radiol 2016;57:98)

Radiology images

Images hosted on other servers:

CT scan of chest



Prognostic factors
  • No significant difference between yolk sac tumor and mixed germ cell tumor
  • Complete surgical resection is the most favorable prognostic factor

Case reports
Treatment
Gross description
  • Solid or cystic with soft pale gray cut surface and gelatinous or mucoid consistency
  • Hemorrhage and necrosis after neoadjuvant therapy
  • Cyst formation may be treatment related or may indicate a mixed germ cell tumor

Gross images

Images hosted on other servers:

Firm, solid white mass



Microscopic (histologic) description
  • Different histologic patterns: microcystic (reticular), macrocystic, glandular - alveolar, endodermal sinus (pseudopapillary), myxomatous, hepatoid, enteric, polyvesicular - vitelline, solid and spindle
  • Loose network of cystic spaces and channels lined by flat or cuboidal cells with scant cytoplasm in a microcystic / reticular pattern
  • Pseudopapillary structures and Schiller-Duval bodies in endodermal sinus pattern (Kaohsiung J Med Sci 2005;21:395)
  • Cysts lined by cuboidal to flat tumor cells surrounded by dense fibrous stroma in a polyvesicular / vitelline pattern (Mediastinum 2019;3:30)
  • Solid pattern resembles seminoma and embryonal carcinoma

Microscopic (histologic) images

Contributed by Sarosh Moeen, M.B.B.S.
Sheet-like growth of cells

Sheet-like growth of cells

Marked pleomorphism

Marked pleomorphism

SALL4

SALL4

OCT3/4

OCT3/4

Glypican 3

Glypican 3



Cytology description
  • Aggregates of medium to large cells with variable nuclei and prominent nucleoli
  • Cytoplasm moderate to abundant
  • Background debris or mucoid material
  • Presence of hyaline globules is typical (Cancer 2008;114:504)

Cytology images

Images hosted on other servers:

Pap; Giemsa



Positive stains
Negative stains
Sample pathology report
  • Anterior mediastinal mass, core biopsy:
    • Germ cell tumor with immunohistochemical profile favoring yolk sac tumor (see comment)
    • Comment: A possibility of mixed germ cell tumor in the vicinity cannot be excluded.

Differential diagnosis
Board review style question #1

A 35 year old man presented with signs and symptoms of superior vena cava syndrome. On radiology, a mass was found in the anterior mediastinum, pushing on to the great vessels. It was resected. The photomicrograph above shows the histologic features. The tumor stained with SALL4 and OCT3/4 and was negative for CD30. What is the most likely diagnosis in this case?

  1. Embryonal carcinoma
  2. Seminoma
  3. Teratoma
  4. Yolk sac tumor
Board review style answer #1
B. Seminoma. The microscopic image shows sheets of tumor cells with fibrous septa infiltrated by lymphocytes. The cells are round to polygonal, uniform epithelioid cells with round to oval nuclei and vesicular chromatin. These features plus the IHC pattern are typical for seminoma. The morphology is not typical for the other germ cell tumors. Answer A is incorrect because embryonal carcinoma is positive for CD30 in addition to the other 2 markers given. Answer C is incorrect because teratoma is negative for all 3 markers. Answer D is incorrect because yolk sac tumor shows only SALL4 positivity. Therefore, the correct answer is seminoma.

Comment Here

Reference: Germ cell tumors
Board review style question #2
A 24 year old man presented with shortness of breath, cough and dysphagia. Radiological examination shows a well demarcated, multilocular cystic mediastinal mass with fat, bone and areas of calcification. On resection, microscopic examination shows cystic cavities lined by stratified squamous keratinized epithelium with adnexal structures underneath. Foci showing undifferentiated mesenchyme, primitive cartilage, neuroectodermal tissue and mature bone are also seen. What is the most likely diagnosis in this case?

  1. Embryonal carcinoma
  2. Immature teratoma
  3. Mature teratoma
  4. Seminoma
  5. Yolk sac tumor
Board review style answer #2
B. Immature teratoma. The radiological findings of the presence of fat, bone and calcification are typical of teratoma. Based on the presence of immature cartilage, mesenchyme and neuroectodermal tissue on microscopy, this represents immature teratoma. Answer C is incorrect because a mature teratoma will not show any such immature components. Answer A is incorrect because embryonal carcinoma does not show derivatives of all 3 germ cell layers; neither do seminoma or yolk sac tumor. Embryonal carcinoma shows uniform population of large polygonal cells with nuclei showing prominent nucleoli, many mitoses and necrosis. Answer D is incorrect because seminoma shows cells divided by incomplete fibrous septae with lymphocytic sprinkling and cells having eosinophilic nucleoli. Answer E is incorrect because yolk sac tumor shows many growth patterns but characteristic endodermal sinus-like formations (also known as Schiller-Duval bodies) can be identified in most cases.

Comment Here

Reference: Germ cell tumors

Grossing & features to report
Table of Contents
Grossing | Features to report
Grossing
  • At least one section of tumor per cm of tumor diameter
  • Tumor and adjacent tissue
  • Tumor invading adjacent tissue
  • Numerous sections of tumor capsule, particularly in areas of apparent disruption
  • Margins
  • Areas with other pathologic findings
  • Other organs or tissues

  • References: policies and procedures for surgeons and pathologists regarding resection specimens of thymic malignancy (J Thorac Oncol 2011;6:S1730)
Features to report
  • Tumor location
  • Tumor size
  • Histologic type
  • Extent of capsular invasion
  • Extent of invasion of adjacent tissue
  • Angiolymphatic invasion
  • Perineural invasion
  • Margin involvement (distance from closest approach of tumor)
  • Lymph node involvement (total number examined, number involved by tumor, size of largest metastasis, presence of extranodal extension of tumor)
  • Additional findings (additional tumors, nonneoplastic tissue)

  • References: recommendations for the reporting of surgically resected thymic epithelial tumors (Hum Pathol 2009;40:918, Am J Clin Pathol 2009;132:10), protocol for the examination of specimens from patients with thymic epithelial tumors located in any area of the mediastinum (Arch Pathol Lab Med 2003;127:1298)


Lymphoepithelial carcinoma
Definition / general
  • Also known as poorly differentiated, nonkeratinizing, squamous cell thymic carcinoma or lymphoepithelioma-like thymic carcinoma
  • Originates from thymic epithelial cells and shows histologic and immunophenotypic evidence of squamous differentiation
  • WHO classification: lymphoepithelial carcinoma of the thymus
Essential features
  • Defined as a primary thymic epithelial neoplasm displaying overt cytologic evidence of malignancy with loss of organotypical features of thymic differentiation
  • Diagnosis of exclusion
  • Invasive growth pattern of nests and cords of tumor cells, which grow in sheets with a syncytial growth pattern and have vesicular nuclei with prominent nucleoli
  • Often have prominent associated lymphoid infiltrate within fibrous stroma
  • Some (~50%) cases are associated with Epstein-Barr virus (EBV) (Cancers (Basel) 2021;13:4794)
Terminology
  • Thymic carcinoma
  • Lymphoepithelial carcinoma
  • Poorly differentiated, nonkeratinizing, squamous cell thymic carcinoma
  • Lymphoepithelioma-like carcinoma
ICD coding
  • ICD-10: C37 - thymic carcinoma (malignant)
Epidemiology
  • M > F (~3:1)
  • Average age of 60 years (range: 20 - 85 years)
Sites
Etiology
Clinical features
  • Presents with anterior mediastinal masses, often found incidentally
  • When symptomatic, clinical symptoms include chest pain, shortness of breath and weight loss (Am J Surg Pathol 2018;42:1224)
Diagnosis
  • Diagnosis of exclusion
  • Requires the presence of a poorly differentiated carcinoma primary to the thymus with an invasive growth pattern of nests and cords of tumor cells with syncytial growth and vesicular nuclei with prominent nucleoli
Radiology description
  • Typically appears as well demarcated anterior mediastinal masses
  • Positron emission tomography (PET) usually shows mild to moderate PET avidity
Prognostic factors
Case reports
Treatment
  • Surgical resection is still the predominant treatment
  • Radiotherapy may increase local regional relapse free survival with mild toxicities in advanced stage patients (Ann Thorac Surg 2017;104:1718)
  • In the largest study, 14 of 20 patients had no evidence of disease between 1.5 and 16 years after diagnosis (median survival: 4 years; mean survival: 5.5 years) (Oncol Lett 2014;8:2183)
Microscopic (histologic) description
  • Invasive growth pattern of cords, nests and trabeculae separated by fibrous stroma with variable prominent mature lymphoid infiltrates
  • 2 patterns of growth described (Am J Surg Pathol 2018;42:1224):
    • Lymphoepithelioma-like pattern: irregular cords and islands of tumor cells separated by connective tissue stroma containing dense lymphoplasmacytic infiltrates
    • Desmoplastic pattern: irregular cords and nests of tumor cells separated by fibrous stroma with minimal inflammatory infiltrates
    • Can have both patterns
  • Can show focal areas of abrupt keratinization
  • Prominent areas of comedonecrosis
  • Uniform, poorly differentiated tumor cell population with round to oval nuclei, vesicular chromatin, prominent round eosinophilic nucleoli and an indistinct rim of eosinophilic cytoplasm
  • Syncytial growth pattern of tumor cells with rare pleomorphic and multinucleated cells
Microscopic (histologic) images

Contributed by David Suster, M.D.

Sheets of large round to oval tumor cells

Anastomosing cords, islands, comedonecrosis

Comedonecrosis

Lymphoepithelioma-like pattern


Desmoplastic pattern

Focal abrupt keratinization

p63

Electron microscopy description
  • Cohesive population of large cells with round to oval nuclei containing a single nucleolus and scattered heterochromatin (Am J Surg Pathol 2018;42:1224)
  • Abundant junctional complexes, with dense plaques and well formed desmosomes containing attached short tonofilaments (Am J Surg Pathol 2018;42:1224)
Electron microscopy images

Contributed by David Suster, M.D.

Large, round tumor cell

2 desmosomal attachments

Molecular / cytogenetics description
Sample pathology report
  • Thymus, resection:
    • Poorly differentiated thymic squamous cell carcinoma, predominantly nonkeratinizing (lymphoepithelial carcinoma) (see synoptic report and comment)
    • Comment: The tumor cells are positive for p40, CD117, CD5 and PDL1.
Differential diagnosis
  • Metastatic poorly differentiated lung carcinoma:
  • Atypical thymoma:
    • Can sometimes have areas suspicious for invasion; however, they are generally well circumscribed with thick fibrous septa (not very inflammatory) and no obvious invasion or necrosis
    • p63+ but CD117 and CD5-
  • Basaloid thymic carcinoma:
    • Less comedonecrosis and basal palisading of the nuclei
    • Lack distinct nucleoli and cleared out vesicular cytoplasm
  • NUT carcinoma:
    • NUT1+
    • NUT::BRD translocations can be identified by FISH
  • Metastatic lymphoepithelial carcinoma:
    • Lymphoepithelial carcinoma may arise in extramediastinal locations as a primary tumor
    • Outside the mediastinum these tumors tend to be strongly correlated with EBV positivity
    • Strict clinical correlation and detailed history required to rule out metastasis
Board review style question #1

A 62 year old patient presented with a 9 cm anterior mediastinal mass (see image above). What would help differentiate this lesion from a metastasis from a head and neck lymphoepithelioma-like thymic carcinoma?

  1. Detailed clinical history
  2. Immunohistochemistry for p63 and cytokeratins
  3. In situ hybridization for EBER
  4. In situ hybridization for HPV
  5. Next generation sequencing
Board review style answer #1
A. Detailed clinical history. Immunohistochemistry and in situ hybridization in general are unhelpful in distinguishing lymphoepithelial carcinomas from different sites. Although a much lower percentage of lymphoepithelial carcinomas test positive for EBV compared to other primary sites, a significant proportion are still reported to express EBV. HPV ISH would not be helpful as it would be expected to be negative. The most useful way to separate these tumors is through a detailed clinical history, including thoracic and head and neck radiologic studies.

Comment Here

Reference: Lymphoepithelial carcinoma
Board review style question #2
Which immunohistochemical stains would be most helpful in distinguishing a primary squamous cell thymic carcinoma from metastatic lesions from other primary sites?

  1. CD5 and CD117
  2. Cytokeratins
  3. p16
  4. p63
Board review style answer #2
A. CD5 and CD117 would be the most helpful in establishing thymic origin in this scenario. While primary lymphoepithelioma-like thymic carcinoma will express keratins, p16 and p63, so would poorly differentiated squamous cell carcinomas elsewhere in the body; however, few lesions outside the body would have CD5 and CD117 positivity.

Comment Here

Reference: Lymphoepithelial carcinoma

Müllerian cyst (Hattori cyst)
Definition / general
Case reports
Gross description
  • 1.3 - 5 cm with thin wall containing smooth muscle
Microscopic (histologic) description
  • Lined by simple cylindrical or cuboidal, nonmucinous and often ciliated epithelium resembling uterine tubal epithelium
Microscopic (histologic) images

Case #416

H&E


PAX8



Images hosted on other servers:

Various images

Positive stains
Negative stains

Mediastinitis, sclerosing
Definition / general
  • Also called idiopathic mediastinal fibrosis, fibrosing mediastinitis
  • Fibroinflammatory lesion, usually anterosuperior mediastinum, often presenting with superior vena cava syndrome or cardiorespiratory compromise
  • All ages
Clinical features
  • Associated with other idiopathic fibrosing conditions such as inflammatory pseudotumor of orbit, retroperitoneal fibrosis, Riedel struma, sclerosing cholangitis
  • Also associated with pulmonary or mediastinal nodal infection due to fungi (Histoplasma), methysergide treatment, phlebitis, syphilis, trauma
Radiology description
  • Asymmetric mediastinal widening with projection of mass into upper lung field
  • Radiologically divided into 2 types: focal (common) and diffuse
    • Focal: localized and calcified mass in paratracheal or subcarinal compartments of mediastinum or in pulmonary hilum
    • Diffuse: diffusely infiltrating, noncalcified mass affecting multiple mediastinal compartments
  • Additional pulmonary findings includes infiltrates, consolidation and pleural effusion
Prognostic factors
  • Prognosis depends mainly on location of fibrosis and structures involved
Case reports
Treatment
  • Steroids, surgical excision
Gross description
  • Tan-yellow, gelatinous masses to gray-white, hard fibrotic masses that compress or infiltrate mediastinal structures
  • Often well demarcated from surrounding tissue
Microscopic (histologic) description
Different patterns:
  • Cellular fibrous reaction with polymorphic inflammatory infiltrate with plasma cells and eosinophils
  • Dense, fibrohyaline tissue with focal spindle cells and inflammatory cells or scattered lymphoid follicles with occasional dystrophic calcification
  • Paucicellular areas with keloid like fibrosis may occur
  • Necrosis and metaplastic bone can occasionally be present (Arch Pathol Lab Med 2010;134:417)

Meningocele-cystic
Definition / general
  • Posterior mediastinum cysts in infants or children which communicate with meninges, usually through a defect in vertebral bodies
  • Contain clear / amber cerebrospinal fluid
  • Usually incidental finding identified prior to surgery
Microscopic (histologic) description
  • Thick fibrous wall, lined by flattened arachnoid cells
  • Variable neural tissue, calcification

Metastases
Definition / general
  • Most mediastinal epithelial neoplasms are metastases, usually from trachea, bronchi, lung, esophagus
  • Usually rapidly fatal
Clinical features
  • Primary lung tumors may present as huge mediastinal mass with occult bronchial primary
Case reports
Microscopic (histologic) images

Case #430

Prostatic adenocarcinoma metastatic to mediastinum

PSA

Stains
Electron microscopy description
  • Premelanosomes suggests melanoma

Myasthenia gravis
Definition / general
  • Defect in nicotinic acetylcholine receptor (AChR) present in subsynaptic membrane of neuromuscular junction (at motor end plate), due to circulating autoantibodies to receptor
  • Acetylcholine receptor also present in normal thymus, in myoid type cells
  • Thymus may contain ectopic germinal centers with B cells producing pathogenic antiacetylcholine receptor antibodies (Ann N Y Acad Sci 2008;1132:135)
Pathophysiology
  • May be due to T cells attacking myoid cells, then T cells induce B cells to produce autoantibodies; physiological connection with thymomas is unclear
  • Two step hypothesis: hyperplastic medullary thymic epithelial cells are involved in provoking infiltration and thymic myoid cells (with intact AChR) are involved in germinal center formation (Am J Pathol 2007;171:893)
  • MG patients have high number / ratio and abnormal distribution of thymic dendritic cells, which may be actively involved in pathogenesis (Zhonghua Yi Xue Za Zhi 2008;88:3349)
  • Autoimmunity may be related to increased toll-like receptor 4 expression in thymus of some myasthenic patients (Am J Pathol 2005;167:129)
Clinical features
  • 12% of myasthenia gravis (MG) patients have other autoimmune diseases, including Graves disease, rheumatoid arthritis
  • MG patients with thymomas may have autoantibodies to titin or other striated muscle antigens
  • 65% of patients have thymic hyperplasia, 25% normal thymus, 10% thymomas; risk factors for thymoma are males with initial MG symptoms age 50+ years
  • Present or develops in 30 - 45% of patients with thymomas, usually months / years after excision of thymoma
  • Lymphoid follicles in thymoma or adjacent thymus indicates higher risk for MG
  • MG associated thymomas are morphologically similar to non-MG associated thymomas
Prognostic factors
Treatment
  • Thymectomy (regardless of presence of thymoma)
Differential diagnosis
  • Lambert-Eaton syndrome:
    • Muscle weakness due to antibodies to neuronal calcium channel

Neuroendocrine carcinoma
Definition / general
  • Similar to neuroendocrine carcinomas at other sites
  • Aggressive neoplasms capable of local recurrence and distant metastasis (Ann Pathol 2005;25:205)
Terminology
  • Atypical carcinoids are considered moderately differentiated neuroendocrine carcinomas
  • Undifferentiated or poorly differentiated neuroendocrine carcinomas may be small cell carcinoma
Clinical features
Case reports
Microscopic (histologic) description
  • Larger cells with more cytoplasm than small cell carcinoma, organoid growth pattern, spindle cell foci
  • May have prominent angiomatoid features (Hum Pathol 1999;30:635)
Microscopic (histologic) images

Images hosted on other servers:

Nests, ribbons and festoons

Various stains

Negative stains
Electron microscopy description
  • Desmosomes, neurosecretory granules

Other nonneoplastic
Acute thymic involution

Definition / general
  • Due to stress (chronic debilitating disease), HIV or other infections, prolonged protein malnutrition and immunosuppressive or cytotoxic drugs, graft versus host reaction
  • Seen in newborn infants with chorioamnionitis and sepsis
  • Thymus size is significantly reduced in preterm infants born to mothers with subclinical, histologically proven chorioamnionitis (Hum Pathol 2000;31:1121)

Microscopic (histologic) description
  • Preservation of lobular architecture and Hassall corpuscles but marked lymphocyte depletion (particularly with HIV)
  • Vessels are large compared to size of lobules
  • Frequent plasma cells, fibrohyaline changes of basement membrane of vessels and thymic epithelium
  • HIV patients also have effacement of corticomedullary junction and inconspicuous Hassall corpuscles

Differential diagnosis
Diffuse thymic fibrosis

Definition / general
  • Uncommon disorder with no / limited symptoms; may have dyspnea, cough or hemoptysis
  • Area of diffuse fibrosis varies from 3.5 - 17 cm, confined to anterior mediastinum
  • Unknown etiology; altered immunity or infection may play a role
  • Males and females, mean age 48 years

Microscopic (histologic) description
  • Diffuse fibrosis with variable collagen deposition, lymphoplasmacytic infiltrates and involution / atrophy of thymus
  • May show IgG4+ plasma cells and focal obliterative phlebitis (Am J Surg Pathol 2010;34:211)
Ectopic thymus

Definition / general
  • Remnants, implants or accessory nodules that may appear from angle of mandible to thyroid gland, most commonly at level of thyroid gland
  • Rarely becomes hyperplastic or neoplastic

Epidemiology
  • Usually an incidental finding during thyroid surgery in preteens, very rare in adults due to thymic involution

Case reports

Microscopic (histologic) description
  • Normal appearing thymic tissue

Microscopic (histologic) images

Images hosted on other servers:

Central cervical epithelial rest

CK5 / 6, p63, CD10



Differential diagnosis
Ectopic tissue in thymus

Definition / general
  • Usually parathyroid tissue or sebaceous glands, rarely thyroid tissue

Case reports
Mediastinitis, acute

Definition / general
  • Usually in posterior mediastinum, due to traumatic perforation of esophagus or descending infection along prevertebral fascia
  • Initial lesion may be a neck abscess
  • Often causes mediastinal abscess which requires surgical drainage
  • Other causes: chest wall infection or postcardiac surgery, often due to CMV

Case reports

Microscopic (histologic) images

Images hosted on other servers:

Extensive acute inflammation

Fibrinoinflammatory
exudate

Mediastinitis, chronic

Definition / general
  • May compress superior vena cava and simulate malignancy
  • Usually anterior to tracheal bifurcation
  • Some cases may represent fibrosing mediastinitis

Microscopic (histologic) description
  • Granulomas, fibrosis; may be fungus, Histoplasma (with thick fibrous capsule), mycobacteria (thin fibrous capsule)
Thymic dysplasia

Definition / general
  • Congenital thymic alteration due to developmental arrest
  • Lack of differentiation of thymic epithelium, responsible for absence of Hassall corpuscles, is main feature (Histopathology 1992;21:499)

Clinical features
  • Associated with severe combined immunodeficiency syndrome, ataxia telangiectasia, chromosomal instability syndromes, Nezelof syndrome (Arch Pathol Lab Med 1987;111:1118)
  • Incomplete form of DiGeorge syndrome is congenital anomaly with a constellation of findings that includes thymic hypoplasia (J Cutan Pathol 2008;35:380); complete form has absent thymus

Gross description
  • Small thymus (< 5 g)

Microscopic (histologic) description
  • Tubules and rosettes of primitive appearing epithelium without segregation into cortical and medullary regions
  • No Hassall corpuscles, no / rare lymphocytes

Differential diagnosis
Thymic follicular hyperplasia

Definition / general
  • Defined as substantial numbers of lymphoid follicles in thymus of adults
  • Thymus usually has normal size / weight

Clinical features
  • Present in 65% with myasthenia gravis
  • Also associated with hyperthyroidism, Addison disease, SLE, early HIV, multilocular cysts, other immune related diseases
  • Often different clinical history than true thymic hyperplasia (Pathologica 2009;101:175)

Microscopic (histologic) description
  • Follicles with germinal centers, medullary epithelial cells may be disordered or hypertrophied

Differential diagnosis
  • Normal lymphoid follicles of infants / children (few present)
True thymic hyperplasia

Definition / general
  • Thymus larger than normal limits for age, based on tables
  • Otherwise histologically unremarkable

Epidemiology
  • Often in infants or children or in adults after cancer chemotherapy

Case reports

Immunohistochemistry

Pericardial cyst
Definition / general
  • Usually at right cardiophrenic angle, adherent to pericardium and diaphragm; may communicate with pericardial cavity
  • Due to failure of one of multiple disconnected lacunae to merge with the others
Clinical features
  • Usually asymptomatic patients of all ages anywhere in mediastinum; rarely associated with dyspnea or chest pain
Radiology description
  • Xray: irregular, adjacent to cardiac contour
Case reports
Gross description
  • Thin fibrous wall, unilocular, collapses when opened, smooth cyst lining, serous fluid contents
Microscopic (histologic) description
  • Fibrous tissue lined by bland mesothelium, rarely with papillary hyperplasia, no cholesterol granulomas, no smooth muscle, no cartilage, no specialized epithelium
Positive stains

Staging-thymic tumors
Definition / general
  • Thymoma, thymic carcinoma, thymic neuroendocrine tumors and combined thymic carcinoma are covered by this staging system
  • First introduced in AJCC 8th edition (Amin: AJCC Cancer Staging Manual, 8th Edition, 2017)
  • AJCC/TNM staging is used for predicting outcomes of thymic tumors, such as recurrence (in the lower stages) and disease specific survival (in the higher stages); reference numbers are available (J Thorac Oncol 2014;9:S65)
  • Pre operative clinical staging is based on physical examination and imaging; post operative clinical staging may be supplemented by pathological findings
  • Stage is determined primarily by levels of local invasion of a thymic malignancy into surrounding mediastinal structures (T classification) while nodal involvement and metastatic spread are much rarer events
  • There is no recommended histologic grading system for thymic tumors
  • Other staging systems also exist (Lung Cancer 2014;83:126)
Essential features
  • AJCC 7th edition staging was sunset on December 31, 2017; as of January 1, 2018, use of the 8th edition is mandatory
  • T, N and M categories are the mainstay for predicting recurrence and survival in patients with thymic tumors
  • Local invasion (T category) is the primary determinant of staging
Primary tumor (T)
  • TX: primary tumor cannot be assessed
  • T0: no evidence of primary tumor
  • T1: tumor encapsulated or extending into the mediastinal fat, may involve the mediastinal pleura
    • T1a: tumor with no mediastinal pleura involvement
    • T1b: tumor with direct invasion of mediastinal pleura
  • T2: tumor with direct invasion of the pericardium (either partial or full thickness)
  • T3: tumor with direct invasion into any of the following: lung, brachiocephalic vein, superior vena cava, phrenic nerve, chest wall or extrapericardial pulmonary artery or veins
  • T4: tumor with invasion into any of the following: aorta (ascending, arch or descending), arch vessels, intrapericardial pulmonary artery, myocardium, trachea, esophagus

Notes:
  • Involvement must be microscopically confirmed in pathological staging, if possible
  • T categories are defined by levels of invasion; they reflect the highest degree of invasion regardless of how many other (lower level) structures are invaded
  • T1, level 1 structures: thymus, anterior mediastinal fat, mediastinal pleura
  • T2, level 2 structures: pericardium
  • T3, level 3 structures: lung, brachiocephalic vein, superior vena cava, phrenic nerve, chest wall, hilar pulmonary vessels
  • T4, level 4 structures: aorta (ascending, arch or descending), arch vessels, intrapericardial pulmonary artery, myocardium, trachea, esophagus
Regional lymph node (N)
  • NX: regional lymph nodes cannot be assessed
  • N0: no regional lymph nodes metastasis
  • N1: metastasis in anterior (perithymic) lymph nodes
  • N2: metastasis in deep intrathoracic or cervical lymph nodes

Note:
  • Involvement must be microscopically confirmed in pathological staging, if possible
Distant metastasis (M)
  • M0: no pleural, pericardial or distant metastasis
  • M1: pleural, pericardial or distant metastasis
    • M1a: separate pleural or pericardial nodule(s)
    • M1b: pulmonary intraparenchymal nodule or distant organ metastasis
AJCC prognostic stage grouping
Stage I: T1a, T1b N0 M0
Stage II: T2 N0 M0
Stage IIIA: T3 N0 M0
Stage IIIB: T4 N0 M0
Stage IVA: any T N1 M0
any T N0, N1 M1a
Stage IVB: any T N2 M0, M1a
any T any N M1b
Board review style question #1
A 62 year old man presented with a large thymic carcinoma directly invading pericardium and no metastatic spread. What is the pT category per the AJCC/TNM 8th edition?

  1. pT1a
  2. pT1b
  3. pT2
  4. pT3
  5. pT4
Board review style answer #1
Board review style question #2
What is the major predictor of thymic carcinoma recurrence?

  1. Distant metastasis
  2. Extent of local invasion
  3. Histological type
  4. Nodal metastasis
  5. Size of tumor
Board review style answer #2
B. Extent of local invasion. T category (local invasion) is a major predictor of recurrence while N and M categories significantly influence survival.

Comment Here

Reference: Staging-thymic tumors

Synovial sarcoma
Definition / general
  • Malignant mesenchymal tumor of uncertain lineage that most often presents in the extremities of young adults
  • While synovial sarcoma occurs in the pleuropulmonary parenchyma with some frequency, it is very rare in the mediastinum (Mod Pathol 2007;20:760)
Essential features
  • Translocation associated sarcoma characterized by SS18::SSX1/2 fusion
  • Monomorphic spindle cell sarcoma with or without areas of epithelial differentiation (monophasic or biphasic, respectively)
  • Accounts for ≤ 10% of thoracic synovial sarcoma cases, with most arising in the lungs or pleura (Mod Pathol 2007;20:760)
Terminology
  • Monophasic synovial sarcoma composed only of spindled cells
  • Biphasic synovial sarcoma composed of spindled and epithelial cells
ICD coding
  • ICD-10:
    • C38.3 - malignant neoplasm of mediastinum, unspecified
    • C49.9 - malignant neoplasm of connective and soft tissue, unspecified
Epidemiology
Sites
  • May involve any mediastinal compartment including anterior (prevascular), posterior (paravertebral), middle (visceral) and superior mediastinum (Am J Surg Pathol 2018;42:761)
Clinical features
  • Typically presents with symptoms related to a large mediastinal mass, which may include pain (chest, back, epigastric, shoulder), chest tightness, dyspnea, cough, dysphagia, hemoptysis, Horner syndrome, hemothorax, obstructive pneumonia or syncope (Am J Surg Pathol 2018;42:761)
  • Constitutional symptoms may also include poor appetite, fatigue, etc. (Am J Surg Pathol 2018;42:761)
Radiology description
  • Usually observed as a lobulated round to oval mass
  • Often reach very large size (> 10 cm) (Am J Surg Pathol 2018;42:761)
  • May compress or encase surrounding critical structures including pericardium, heart or great vessels
Radiology images

Contributed by Jennifer M. Boland, M.D.

CT scan

Prognostic factors
  • Seem to be particularly aggressive with a worse prognosis than soft tissue counterparts
    • ~70% of patients die from disease < 3 years from diagnosis (Am J Surg Pathol 2018;42:761)
    • Aggressive clinical course is probably due to site specific factors, such as proximity to critical anatomic structures as well as late detection with large tumor size at diagnosis
    • Adverse outcome may also be related to the high rate of poorly differentiated morphology
  • No known differences are noted in the clinical behavior of monophasic and biphasic synovial sarcoma; therefore, this distinction is not important for clinical management purposes
  • Poorly differentiated morphology is associated with older age at presentation and poor prognosis
Case reports
Treatment
  • Surgery with or without adjuvant chemotherapy / radiotherapy is the treatment of choice, if feasible
  • For patients with inoperable tumors, chemotherapy and radiation are typically employed (Am J Surg Pathol 2018;42:761)
Gross description
  • Average tumor size is large, ~13 cm (range 5 - 23 cm) (Am J Surg Pathol 2018;42:761)
  • Tumors are grossly tan-white, soft to rubbery cut surface with hemorrhage and necrosis
  • May show cystic degeneration and gelatinous change
  • Often appear grossly well circumscribed / lobulated but usually show subtle invasive growth into adjacent structures, including chest wall, great vessels, heart, pericardium, pleura, ribs or vertebra (Mod Pathol 2007;20:760, Am J Surg Pathol 2005;29:569)
Microscopic (histologic) description
  • Can show monophasic or biphasic morphology but monophasic is more common in the mediastinum (Am J Surg Pathol 2018;42:761, ISRN Oncol 2014;2014:412527, Mod Pathol 2007;20:760, Am J Surg Pathol 2005;29:569)
    • Monophasic
      • Consist solely of monomorphic spindled cells with nuclear monotony; marked pleomorphism is usually absent
      • Mitotic activity and necrosis are common
      • Orderly fascicular / herringbone growth pattern
      • Hemangiopericytoma-like vascular pattern
      • Hyalinized collagen and calcifications may be present
    • Biphasic
      • Comprised of variable proportions of monomorphic spindled cells as described above and epithelial cells
      • Epithelial cells can form glands or solid nests and cords; seldom show squamous metaplasia or granular cell change
    • Poorly differentiated
      • Form of tumor progression with some deviation from classical morphology; areas of conventional morphology are usually also present
      • Poorly differentiated areas are very cellular and may be composed of primitive round cells, epithelioid cells or high grade spindled cells
      • Usually demonstrate elevated mitotic activity, frequent rhabdoid change and necrosis
Microscopic (histologic) images

Contributed by Jennifer M. Boland, M.D.

Monophasic synovial sarcoma

EMA

TLE1


Biphasic synovial sarcoma

Poorly differentiated synovial sarcoma

Virtual slides

Images hosted on other servers:

Monophasic synovial sarcoma

Positive stains
  • SS18::SSX and SSX C terminus (Histopathology 2020;77:588, Am J Surg Pathol 2020;44:922)
    • Immunohistochemistry with antibodies directed against the SS18::SSX fusion protein and the C terminus of SSX are sensitive and specific markers of synovial sarcoma, especially when performed together
    • If positive in the correct morphological context, molecular confirmation is usually not required
  • Pankeratins and EMA (Virchows Arch 2000;437:275):
    • Usually very limited expression in the spindle cell component
    • Often restricted to single cells or sometimes highlights glandular structures that are unapparent on H&E
    • Glandular areas of biphasic examples are usually robustly positive, if present
    • May be completely negative if only the spindle cell component is present, especially on small biopsies
    • Spindle cell areas generally do not show diffuse immunoreactivity (in contrast to sarcomatoid carcinomas, mesotheliomas and thymomas)
  • TLE1 (Am J Clin Pathol 2011;135:839):
    • Highly sensitive nuclear marker of synovial sarcoma
    • Not specific, since other tumors can show staining
    • Most valuable to identify cases in which molecular genetic testing is likely to be positive (i.e. a negative stain is a good "rule out" marker but positive stain should be confirmed by molecular studies if the diagnosis is in doubt)
  • CD99 (Mod Pathol 2006;19:659):
    • Often positive but poor specificity as other entities in differential diagnosis (i.e. Ewing sarcoma) also express this marker
  • S100 (Mod Pathol 2006;19:659):
Molecular / cytogenetics description
  • In the absence of SS18::SSX and SSX C terminus immunohistochemistry, definitive diagnosis often requires ancillary molecular genetic testing due to the rare primary site; these tests are possible even on small biopsies
  • t(X;18)(p11;q11)
Molecular / cytogenetics images

Contributed by Jennifer M. Boland, M.D.

SS18(SYT)-SSX1 fusion

Differential diagnosis
Board review style question #1
A 37 year old man presented with shortness of breath and dull chest pain. He was found to have a 15 cm mass in the anterior mediastinum. Representative photomicrographs from the needle biopsy are shown below. What is the most likely molecular alteration in this tumor?



  1. EWSR1::FLI1
  2. FUS::DDIT3
  3. NAB2::STAT6
  4. SS18::SSX1
  5. WWTR::CAMTA
Board review style answer #1
D. SS18::SSX1. t(X;18)(p11;q11) is characteristically seen in synovial sarcoma and leads to the fusion of SS18 (also known as SYT or SST) with one of the SSX genes. It can be detected by next generation sequencing, RT-PCR or FISH. SYT::SSX1 is the most common fusion in synovial sarcoma, present in 67% of cases. The tumors characterized by the other fusion genes listed would not be expected to show the cellular monomorphic spindle cell sarcoma observed in the photomicrograph. EWSR1::FLI occurs in Ewing sarcoma; FUS::DDIT3 is observed in myxoid liposarcoma; NAB2::STAT6 fusion characterizes solitary fibrous tumor; and WWTR::CAMTA fusion is present in epithelioid hemangioendothelioma.

Comment Here

Reference: Synovial sarcoma
Board review style question #2
Which of the following features essentially excludes a monophasic synovial sarcoma from the differential diagnosis in a mediastinal needle biopsy of a spindle cell tumor?

  1. Focal CD99 expression
  2. Focal S100 expression
  3. Lack of keratin expression
  4. Marked nuclear pleomorphism
  5. Small round blue cell morphology
Board review style answer #2
D. Marked nuclear pleomorphism. Monophasic synovial sarcomas are spindle cell sarcomas with striking nuclear monotony and generally should not show marked nuclear pleomorphism, even in poorly differentiated examples. The lack of keratin expression, especially on small biopsies, does not exclude the diagnosis if the correct morphology is observed and molecular confirmation could be considered in this context. Focal S100 expression can be observed in synovial sarcoma and in this setting, molecular analysis may be needed to differentiate it from malignant peripheral nerve sheath tumor. Poorly differentiated examples of synovial sarcoma can show small round blue cell morphology, often resembling Ewing sarcoma, which also may require molecular analysis since both entities can express CD99.

Comment Here

Reference: Synovial sarcoma

Teratoma-cystic
Definition / general
  • Common; 10 - 20% of mediastinal lesions
  • In anterosuperior mediastinum
  • Neoplastic, not a developmental malformation
  • Usually contain tissue derived from at least 2 of 3 germ cell layers - endoderm, mesoderm, ectoderm
  • 50% have cough, dyspnea or chest pain
  • Tumors adhering to lung, pericardium or blood vessels are either malignant or ruptured mature teratomas with inflammatory reaction
Epidemiology
  • Usually children or young adults (mean age 20 years)
Clinical features
  • May be associated with Klinefelter syndrome (XXY)
  • Rarely associated with acute myelogenous leukemia
Case reports
Gross description
  • Cystic with variable solid component
  • Malignant tumors are often adherent to adjacent structures or have necrosis
  • Benign tumors often have keratinous debris, cartilage or mucus
Microscopic (histologic) description
  • Mature squamous epithelium with cutaneous appendages (commonly), GI columnar epithelium, neuroglia, bone, cartilage, fat, striated muscle
  • Also choroid plexus, hepatocytes, pancreas, retinal type tissue
  • May contain immature neuroepithelial tissue (see Immature teratoma below)
  • No cholesterol granulomas
Microscopic (histologic) images

Images hosted on other servers:

Mature cystic teratoma

Mature teratoma

Immature teratoma
Definition / general
  • Contains immature neuroepithelial tissue such as embryonic tubules
  • Benign behavior in patents under age 15
Mature teratoma with malignant transformation
Definition / general
  • Very rare
  • Contain overtly malignant tissue
  • Tumors with germ cell components are classified as malignant mixed germ cell tumors

Case reports

Thymic carcinoma-other variants
Adenocarcinoma
Definition / general
  • Rare, < 10 cases reported
  • Most common type is papillary, mucinous subtype also reported

Epidemiology
  • Mean age 50 years but wide range

Clinical features
  • Must rule out metastatic tumor
  • More likely to be thymic primary if CD5+ and transition from thymoma or thymic cyst to adenocarcinoma is present
  • Outcome is highly variable

Case reports
Basaloid
Definition / general
  • Primary thymic tumor or metastatic tumor to mediastinum from oropharynx, hypopharynx, larynx, esophagus, lung, anorectum

Clinical features
  • Present with intrathoracic mass
  • Primary tumors are associated with multilocular thymic cysts
  • Capable of aggressive behavior and significant mortality (Am J Surg Pathol 2009;33:1113)

Prognostic factors
  • Good prognosis with few fatalities if well differentiated

Microscopic (histologic) description
  • Nests, cords and sheets of small polygonal cells with minimal cytoplasm, hyperchromatic round nuclei, abundant mitotic figures
  • No nuclear molding
  • Separated by myxoid or eosinophilic stroma
  • May have peripheral pallisading of nuclei
  • May have areas of squamous differentiation with keratin pearls or stromal mucin containing gland-like profiles
  • May have cystic structures

Microscopic (histologic) images

Images hosted on other servers:

Prominent peripheral palisading



Positive stains

Negative stains
  • Neuroendocrine markers

Electron microscopy description
  • Poorly differentiated squamous proliferation with limited cytoplasmic tonofilaments, well formed desmosomes, redundant basal lamina
  • No neurosecretory granules
Clear cell
Definition / general
  • Rare; clear cytoplasm due to either glycogen or degenerative changes
  • Almost always fatal

Case reports

Microscopic (histologic) description
  • Lobular or occasionally sheet-like growth pattern of polygonal cells with clear cytoplasm, rather bland nuclear features, round vesicular nuclei, prominent nucleoli; delicate fibrovascular stroma (Am J Surg Pathol 1995;19:835)
  • No blood lakes, no mucin

Microscopic (histologic) images

Images hosted on other servers:

Large cells with abundant optically clear cytoplasm



Positive stains

Differential diagnosis
Mucoepidermoid
Clinical features

Case reports

Microscopic (histologic) description
  • Sheets, lobules and nests of squamous, mucinous and intermediate cells in densely fibrotic stroma

Microscopic (histologic) images

Images hosted on other servers:

Well differentiated mucoepidermoid carcinoma of the thymus

Poorly differentiated
mucoepidermoid
carcinoma of the
thymus

Sarcomatoid
Definition / general
  • Very rare; usually presents as rapidly growing, anterior mediastinal mass
  • Must exclude metastases before making this diagnosis
  • May arise from malignant transformation of preexisting spindle cell thymoma (Am J Surg Pathol 1999;23:691)
  • Aggressive: recurrence, metastasis and death are common

Terminology
  • Also called spindle cell thymic carcinoma

Case reports

Gross description
  • Large (mean 15 cm), well circumscribed, locally infiltrative
  • Firm cut surface, focal hemorrhage, necrosis, cystic changes

Microscopic (histologic) description
  • Irregular fascicles of fusiform and pleomorphic cells with amphophilic or eosinophilic cytoplasm, hyperchromatic nuclei, prominent nucleoli (at least focally), brisk mitotic activity, often atypical mitotic figures
  • May have epithelioid foci
  • Usually transitional areas with spindle cell thymoma
  • May also have lymphoepithelioma-like or anaplastic areas
  • Rarely rhabdomyogenic foci with cross striations

Microscopic (histologic) images

Images hosted on other servers:

Atypical population
of spindle cells
with frequent
mitotic figures



Positive stains

Negative stains

Electron microscopy description
  • Focal junctional complexes between spindle cells, tonofibrils

Differential diagnosis
Undifferentiated
Definition / general
  • Undifferentiated large cell thymic carcinoma accompanied by inflammatory reaction resembling hyaline vascular Castleman disease (Am J Surg Pathol 2005;29:490)
  • May be asymptomatic with incidental detection
  • Tumor cells are cytokeratin+ / CD5-
  • May have neuroendocrine features
  • Indolent behavior

Thymic cyst
Definition / general
  • Thymus derived from third and fourth branchial pouch, as is parathyroid gland
  • Usually presents as incidental mass in anterosuperior mediastinum
  • Congenital (unilocular) or acquired (multilocular)
  • Rarely occur postoperatively
  • Mixed multilocular thymic cyst: has parathyroid or salivary gland tissue
Epidemiology
  • Usually ages 20 - 50 years
Clinical features
Radiology description
  • Xray: rounded, circumscribed masses in anterior mediastinum, may have peripheral rim of calcification
Case reports
Gross description
  • ≤ 18 cm
  • Unilocular with thin wall and serous fluid or multilocular with turbid, cheesy or hemorrhagic material, thick wall and fibrous adhesions
  • Either centered in thymus or connected to it by a small pedicle
Gross images

Case #333


Images hosted on other servers:

Thin walled cyst with thyroid

Microscopic (histologic) description
  • Unilocular cysts:
    • Have thin wall with a few layers of bland squamoid cells and thymic tissue in wall, no inflammation, no cholesterol granulomas, no hemorrhage
  • Multilocular cysts:
    • May have more layers of squamoid, cuboidal, columnar, micropapillary or mixed glandular epithelium
    • May have pseudoepitheliomatous hyperplasia
    • Usually cholesterol granulomas
    • Commonly lymphocytes, granulation tissue, hemorrhage
    • Cysts separated by thick fibrous septae
    • 50% have Hassall corpuscles or other thymic tissue but not in cyst wall
    • No cartilage or smooth muscle is present
Microscopic (histologic) images

Case #333



Various images



Images hosted on other servers:

Bland squamoid epithelium

Papillary outpouchings

Cholesterol clefts

Proliferating multilocular thymic cyst
Definition / general
  • Resembles cutaneous proliferating epidermoid cyst and proliferating trichilemmal cyst

Microscopic (histologic) description
  • Pseudoepitheliomatous hyperplasia of cyst lining cells (narrow tongues of squamoid epithelium extending deeply into fibrous cyst wall) with reactive changes but no dysplasia
  • Typical mitotic figures present

Differential diagnosis

Thymic follicular hyperplasia
Definition / general
  • Refers to the increased number of lymphoid follicles within the thymic gland
Essential features
  • Usually found incidentally or could present with systemic symptoms (StatPearls: Thymic Hyperplasia [Accessed 28 February 2023])
  • Associated with autoimmune diseases such as myasthenia gravis
  • Defined as increased number of lymphoid follicles with retained thymic architecture regardless of the gland size
Terminology
  • Thymic lymphoid hyperplasia
  • Thymic germinal center hyperplasia
ICD coding
  • ICD-10: E32.0 - persistent hyperplasia of thymus
Epidemiology
  • May be idiopathic or seen in association with other autoimmune disorders, including myasthenia gravis, Graves disease, systemic lupus erythematosus, rheumatoid arthritis, scleroderma and allergic vasculitis
Sites
  • Anterior mediastinum
Pathophysiology
Etiology
  • Caused by stimulation of B cell compartment and by abnormal growth of secondary follicles
Clinical features
Diagnosis
  • Usually discovered incidentally for an unrelated chest complaint or systemic symptom
  • Thymic gland usually retains its normal shape but it may be either of normal size or enlarged
  • May be associated with autoimmune diseases such as myasthenia gravis and Graves disease
Laboratory
  • Acetylcholine receptor antibodies may be positive in patients with myasthenia gravis
  • Increased T3 / T4, decreased TSH, increased uptake of radioactive iodine and positive thyroid receptor antibodies in patients with Graves disease
  • Possibly workup for other autoimmune disease if clinically suspicious
  • References: StatPearls: Graves Disease [Accessed 19 June 2023], JAMA Neurol 2015;72:1170
Radiology description
  • Suggested on chest Xray, CT and possibly MRI
  • Findings can range from subtle mediastinal findings to large mass or widened mediastinum
  • Reference: Cancer Imaging 2005;5:139
Radiology images

Images hosted on other servers:

Incidental thymic follicular hyperplasia

Prognostic factors
  • Overall, good prognosis
Case reports
Treatment
  • Asymptomatic patients can be followed up
  • Management of thymic hyperplasia depends on the underlying disease and signs and symptoms
    • Myasthenia gravis: acetylcholinesterase inhibitors, corticosteroids, plasma exchange therapy, thymectomy
    • Graves disease: often regresses after treatment of Graves disease
  • Patients with serious manifestations such as superior vena cava syndrome or myasthenia gravis should be referred for thymectomy
  • Reference: StatPearls: Thymic Hyperplasia [Accessed 19 June 2023]
Gross description
  • Thymus gland may or may not be associated with overall gland enlargement
Gross images

Images hosted on other servers:

Pathogenesis associated with rheumatoid arthritis

Frozen section description
  • Thymus gland with or without discrete mass
  • No evidence of invasion
  • Retained thymic architecture, increased number of lymphoid follicles with reactive germinal centers
  • Presence of Hassall corpuscles
  • Absence of a thick fibrous capsule and fibrous septa
Microscopic (histologic) description
Microscopic (histologic) images

Contributed by Azadeh Esmaeili, M.D. and Sarmad H. Jassim, M.D.
Increased lymphoid follicles

Increased lymphoid follicles

Germinal centers of different sizes

Germinal centers of different sizes

Germinal centers with tingible body macrophages

Germinal centers with tingible body macrophages

Frequent Hassall corpuscles

Frequent Hassall corpuscles

Positive stains
Negative stains
Sample pathology report
  • Thymus, thymectomy:
    • Thymic follicular hyperplasia (see comment)
    • Comment: Thymic tissue, consisting of thymic epithelial cells and associated small mature lymphocytes, in addition to occasional lymphoid follicles with prominent germinal centers. Hassall corpuscles are also noted. These histopathologic findings represent thymic follicular hyperplasia, a condition present in association with myasthenia gravis. Defining morphologic features of thymoma are not present.
Differential diagnosis
  • True thymic hyperplasia:
    • Expansion of the normal component of the thymic gland, normal cortex, medulla, Hassall corpuscles in an organoid configuration
  • Thymoma:
    • Thymic follicular hyperplasia is discerned from thymoma by the preservation of thymic architecture in thymic lymphoid hyperplasia
  • Lymphoproliferative disorders involving the thymus (e.g., T lymphoblastic lymphoma / leukemia, B and T cell lymphomas, classic Hodgkin lymphoma):
    • Effaced architecture in lymphoma versus an expansion of the normal component of the thymic gland in true thymic hyperplasia
  • Germ cell tumor (e.g., seminoma) with florid follicular lymphoid hyperplasia:
    • Usually morphology is different; however, seminoma can have lymphoid hyperplasia and remnant thymic tissue
  • Normal thymus with rare germinal centers
  • Thymic carcinoma:
    • Distorted architecture with desmoplasia and often increased cytologic atypia
Board review style question #1

Which of the following is a feature seen in thymic follicular hyperplasia?

  1. Diffuse meshwork of epithelial cells
  2. Effacement of normal thymic architecture
  3. Lack of reactive secondary follicles
  4. Large lobules separated by fibrous bands
  5. Normal distribution of cortex and medulla
Board review style answer #1
E. Normal distribution of cortex and medulla. Answers A - D are incorrect because effacement of normal thymic architecture, large lobules separated by fibrous bands, lack of reactive secondary follicles and diffuse meshwork of epithelial cells are features of thymoma.

Comment Here

Reference: Thymic follicular hyperplasia
Board review style question #2
Which of the following is the most common thymic entity found in patients with myasthenia gravis?

  1. Lymphoma
  2. Thymic follicular hyperplasia
  3. Thymoma
  4. True thymic hyperplasia
Board review style answer #2
B. Thymic follicular hyperplasia. Thymic follicular hyperplasia may be seen in association with autoimmune disorders, including myasthenia gravis, Graves disease, systemic lupus erythematosus, rheumatoid arthritis, scleroderma and allergic vasculitis.

Comment Here

Reference: Thymic follicular hyperplasia

Thymic squamous cell carcinoma
Definition / general
  • By definition, has overt cellular anaplasia
Epidemiology
  • Ages ≥ 50 years; occasionally children
Diagrams / tables

Images hosted on other servers:

Proposed stage T1, tumor limited to thymic gland

Proposed stage T2, tumor invades nearby structures

Proposed stage T3, direct (continuous) extrathoracic tumor extension beyond thoracic inlet or below diaphragm

Clinical features
  • Associated with hypercalcemia, elevated parathyroid hormone levels, pulmonary sarcoidosis
  • Not associated with paraneoplastic syndromes such as myasthenia gravis or pure red cell aplasia
  • Patients usually present with mass related symptoms
  • Aggressive clinical course
  • Must exclude other primaries, which are much more common (lung, trachea, bronchi, esophagus)
  • Usually squamous cell carcinoma and variants (lymphoepithelioma-like, basaloid)
  • Proposed staging system (Am J Clin Pathol 2012;138:115)
Radiology images

Contributed by Lina Hu, M.D. (Case #479)

Mass with irregular, infiltrative borders

Prognostic factors
  • May be less aggressive than commonly believed; important prognostic factors are lymph node status and tumor size (Am J Clin Pathol 2012;138:103)
  • Keratinizing: good prognosis with few fatalities if well differentiated
Case reports
Gross description
  • Unencapsulated, no internal fibrous septation, firm / hard / gritty with gray-white cut surface, necrosis and hemorrhage
Microscopic (histologic) description
  • Usually cohesive cellular growth, regularly round / oval nuclear outlines, eosinophilic nucleoli, geographic necrosis
  • Usually foci of medullary differentiation, abortive Hassall corpuscles, rosettes, gland-like spaces, T lymphocytes; no perivascular spaces
  • Keratinizing:
    • Similar to tumor in skin, lung, other sites
    • Lobular growth with fibrous bands
    • Nests and cords of large polyhedral cells with intercellular bridges
    • Vesicular or hyperchromatic nuclei, prominent nucleoli, eosinophilic or glassy cytoplasm, keratin pearls
    • Also angiolymphatic invasion, necrosis
    • Rarely coexists with thymoma
  • Non-keratinizing
    • Angular nests of malignant squamous cells in desmoplastic stroma
    • No intercellular bridges, no eosinophilic cytoplasm, no keratin pearls
Microscopic (histologic) images

Contributed by Lina Hu, M.D. (Case #479)

H&E

KIT / CD117

p63

Positive stains
Negative stains
Electron microscopy description
  • Well formed desmosome-like intercellular junctions, cytoplasmic tonofilaments that may insert into junctional complexes
Differential diagnosis
Board review style question #1
Which 2 antibodies are most helpful in diagnosing thymic carcinoma?

A. CK5/6, 34bE12
B. CD5, p63
C. CEA, synaptophysin
D. p63, p40
E. KIT / CD117, INSM1
Board review style answer #1
B. CD5 combined with p63 are helpful thymic markers.

Comment Here

Reference: Thymic squamous cell carcinoma

Thymolipoma
Definition / general
  • Increased thymic volume, due to lobules of mature adipose tissue mixed with unremarkable thymic tissue
  • Benign
  • May be neoplasm of thymic fat (Ann Diagn Pathol 2009;13:185)
Epidemiology
  • Usually young to middle aged adults, found incidentally
Clinical features
  • 10% associated with thymoma-like paraneoplastic symptoms
Case reports
Gross description
  • Encapsulated, up to 20 cm
  • Resembles lipoma
Microscopic (histologic) description
  • Thin fibrous capsule surrounding lobules of mature adipose tissue intimately associated with unremarkable thymic tissue containing cortex, medulla and Hassall corpuscles
  • May have thymic epithelial proliferation, myoid cells, zones of dense fibrosis
Microscopic (histologic) images

Images hosted on other servers:
Missing Image Missing Image

Mature adipose tissue admixed with unremarkable thymic tissue


Thymoma
Definition / general
  • Malignant epithelial tumor of the thymic gland
Essential features
  • Malignant
  • Lobulated architecture comprised of cellular lobules intersected by fibrous bands
  • Neoplastic cells: epithelial cells
  • Thymocytes are reactive
  • Prognostic parameters:
    • Stage
    • Completeness of resection
    • Morphology (as defined by WHO)
Terminology
  • WHO type B3 (by some authors referred to as atypical thymoma)
  • Anterior (prevascular) mediastinum (J Thorac Oncol 2014;9:S97)
  • Middle (visceral) mediastinum
  • Posterior (paravertebral) mediastinum
ICD coding
  • ICD-11:
    • 2C27.2 - malignant neoplasm of thymus
    • XH3734 - thymoma, NOS
    • XH6WN9 - WHO type A thymoma
    • XH0JH0 - WHO type AB thymoma
    • XH66U8 - WHO type B1 thymoma
    • XH2G89 - WHO type B2 thymoma
    • XH4EW9 - WHO type B3 thymoma
    • XH56K5 - micronodular thymoma with lymphoid stroma
    • XH3DX0 - metaplastic thymoma
Epidemiology
  • Most common solitary lesion in mediastinum and prevascular (anterior) mediastinum in adults (J Thorac Oncol 2020;15:568)
  • 1 - 5 per million population per year in adults
  • Any age, peak 40 - 60 years old (Mayo Clin Proc 1993;68:1110, J Thorac Oncol 2015;10:691)
  • M = F
  • 34 - 40% associated with paraneoplastic / autoimmune syndromes
    • Myasthenia gravis (MG)
    • Other neuromuscular syndromes / diseases (Lambert-Eaton syndrome, myotonic dystrophy)
    • Autoimmune diseases (systemic lupus erythematosus, polymyositis, myocarditis, Sjögren syndrome, ulcerative colitis)
    • Endocrine disorders (Addison disease, hyperthyroidism)
    • Hematologic diseases (hypogamaglobulinemia, red cell aplasia, pancytopenia), aplastic anemia
    • Others (hypertrophic pulmonary osteoarthropathy) (Mayo Clin Proc 1993;68:1110)
    • Patients with thymoma and paraneoplastic / autoimmune syndrome: more commonly younger, female, smaller tumors, B2 or B3 subtype, lower stage, R0 resection (J Thorac Oncol 2018;13:436)
Sites
Etiology
  • Tumor cells originate from thymic epithelial cells
Diagrams / tables
pTNM staging for thymic epithelial tumors (8th edition of the AJCC / UICC staging)
 Stage  T  N  M  Tumor Extension
 I   1a
 1b 
 0   0   Encapsulated or extending into mediastinal fat, no involvement of mediastinal pleura
 Invasion of mediastinal pleura 
 II   2   0   0   Invasion of pericardium 
 IIIA   3   0   0   Invasion into lung, brachiocephalic vein, superior vena cava, chest wall, phrenic
 nerve or extrapericardial pulmonary artery or veins 
 IIIB   4   0   0   Invasion into aorta (ascending, arch, or descending), arch vessels, intrapericardial
 pulmonary artery, myocardium, trachea, esophagus 
 IVA   Any   1
 0, 1 
 0
 1a 
 Anterior (perithymic) lymph nodes
 Separate pleural / pericardial nodule(s) (implants) 
 IVB   Any   2
 Any 
 0, 1a
 1b 
 Deep intrathoracic / cervical lymph nodes
 Pulmonary intraparenchymal nodule, distant organ metastasis 


Recommended reporting of thymic epithelial tumors with multiple components (2021 WHO)
 Thymoma with > 1 histologic pattern
 (except AB thymoma) 
 List all thymoma subtypes, including
 percentages in 10% increments; start with
 predominant component 
 Thymoma and thymic carcinoma   List all components, including % in 10%
 increments; start with thymic carcinoma
 component independent of extent 
 Thymoma and thymic carcinoid tumor   List all components, including % in 10%
 increments; start with thymic carcinoid tumor
 component independent of extent 
Reference: Amin: AJCC Cancer Staging Manual, 8th Edition, 2017
Classification
Classification of thymomas according to 2021 WHO
 Type A thymoma (including atypical subtype) 
 Type AB thymoma 
 Type B1 thymoma 
 Type B2 thymoma 
 Type B3 thymoma 
 Micronodular thymoma with lymphoid stroma 
 Metaplastic thymoma 
 Lipofibroadenoma 
Clinical features
Diagnosis
  • Suggested on chest Xray, confirmed by CT; possibly MRI
  • Preresection biopsy usually not performed or necessary
Laboratory
  • None for thymoma
  • Acetylcholine receptor binding antibodies if myasthenia gravis
  • Possibly workup for other paraneoplastic / autoimmune syndromes if clinically suspicious
Radiology description
  • Chest Xray: usually ovoid or lobulated, smooth, well marginated mass, projecting over the mediastinum, more commonly protruding unilaterally (J Thorac Oncol 2010;5:S296)
  • Chest CT: typically well defined, round or lobulated, homogeneous lesion that enhances after contrast injection
    • Can be heterogeneous or cystic
    • Used to evaluate for invasion
  • MRI increasingly utilized, specifically in differentiating thymoma from thymic cyst or thymic hyperplasia, to identify phrenic nerve involvement or to evaluate for invasion in patients with contraindication to iodinated contrast (Magn Reson Imaging Clin N Am 2015;23:165, J Cancer 2019;10:3208)
Radiology images

Contributed by Anja C. Roden, M.D.
WHO type A thymoma

WHO type A thymoma

Prognostic factors
  • Stage (TNM stage; previously Masaoka-Koga stage), in some studies independent of resection status and WHO subtype (J Thorac Oncol 2015;10:691)
  • Complete resection associated with overall survival
  • WHO subtype associated with overall and disease free survival
  • Size of thymoma, age, weight loss are associated with outcome in some studies
Case reports
Treatment
  • Thymectomy with or without resection of adjacent structures depending on extent of tumor, whenever possible
  • Possibly neoadjuvant chemotherapy or radiation or postoperative chemotherapy or radiation, depending on stage and involvement of margins
  • Immune checkpoint inhibitors: prospective trials have shown durable responses and appears to improve survival in patients with relapsed thymoma
    • Controversial, possibly contraindicated; severe and life threatening immune toxicity can occur and therefore extreme caution is necessary while using immunotherapy in patients with thymoma
    • Currently, this treatment is only recommended in the setting of clinical trials with close monitoring (Mediastinum 2021;5:23)
  • Targeted therapies: only limited activity (Lung Cancer 2018;126:25)
Gross description
  • Lobulated
  • Gray-tan
  • Often encapsulated
  • Possibly with cystic changes
  • Sampling of perithymic lymph nodes important for staging
Gross images

Contributed by Anja C. Roden, M.D.
Gross specimen of thymoma

Gross specimen of thymoma

Frozen section description
  • Lobulated neoplasm with fibrous bands and cellular lobules
  • Sometimes only a few fibrous bands present
  • Cellular lobules comprised of (i) a mixture of lymphocytes and large polygonal (epithelial neoplastic) cells, (ii) polygonal cells only without or only with a few lymphocytes or (iii) bland short spindle cells
  • Associated cyst(s) possible
  • Evaluation of margins: discussion with surgeon important to identify true surgical margin as not the entire specimen surface is considered surgical margin
  • Invasion into adjacent structures (mediastinal pleura, pericardium, phrenic nerve, lung, great vessels, among others)
Frozen section images

Contributed by Anja C. Roden, M.D.
Frozen section images of thymoma

Lobulated architecture

Abundant lymphocytes

Abundant lymphocytes

Hassall corpuscle like elements

Hassall corpuscle-like elements

Surgical margin negative

Surgical margin negative

Microscopic (histologic) description
  • Low magnification: lobulated architecture with cellular lobules and intersecting fibrous bands; usually at least partially encapsulated
  • High magnification: neoplastic epithelial cells (polygonal or spindled), various numbers of lymphocytes (thymocytes)
  • Sometimes associated with cyst(s) or focal cystic changes
  • Thymomas may be almost entirely necrotic, sclerosed or ossified
  • Morphologic classification according to 2021 WHO
    • Letters based on shape of neoplastic cells (A spindled, B polygonal)
    • Numbers in type B thymoma (B1 → B3) based on:
      • Increase in ratio of neoplastic cells to thymocytes
      • Increase in cytologic atypia of neoplastic cells
      • Medullary islands present in B1; occasional in B2; absent in B3
    • Type A: bland oval / spindled cells
      • Rare or no thymocytes
      • Various patterns: hemangiopericytoma-like vascular pattern, rosettes / pseudorosettes, microcystic changes
    • Atypical A
      • Same as A and focal
        • Hypercellularity or
        • Increased mitotic count or
        • Coagulative tumor necrosis
    • Type AB: morphology of A + B1 or A + B2, components can be intermingled or separated
    • Type B1: medullary islands (Hassall corpuscle-like elements, paler areas), scattered neoplastic cells (< 3 contiguous epithelial cells)
    • Type B2: mixed neoplastic cells and thymocytes, medullary islands occasionally present
    • Type B3: neoplastic cells
      • Rare or no thymocytes
      • Perivascular spaces with palisading of neoplastic epithelial cells are common (although not specific to type B3 thymomas)
    • Subtyping not recommended on biopsies because of potential heterogeneity
  • Uncommon thymomas:
    • Micronodular thymoma with lymphoid stroma:
      • Nodular appearance due to demarcated nodules or interlacing strands of neoplastic cells that are reminiscent of those of type A thymoma
      • Background predominantly B cells with scattered secondary follicles
      • Only scattered thymocytes
    • Metaplastic thymoma:
      • Biphasic
      • Neoplastic cells reminiscent of type A thymoma
      • Stromal spindled cells
      • YAP::MAML2 rearrangement (Mod Pathol 2020;33:560)
    • Entities that have been obsoleted in the 2021 WHO: microscopic thymoma (small thymic epithelial cell nests unlikely to be neoplastic; do not appear to be precursors of thymoma), sclerosing thymoma (appears to be sclerotic change of existing thymoma rather than distinct form of thymoma)
  • Lipofibroadenoma:
    • Resembles fibroadenoma of the breast
    • Fibrotic and hyaline stroma, focal adipose tissue, strands of bland appearing epithelial cells and a few lymphocytes
    • May contain Hassall corpuscles or calcifications
    • Considered benign
  • Reference: Semin Diagn Pathol 2022;39:99, Adv Anat Pathol 2021;28:291
Microscopic (histologic) images

Contributed by Anja C. Roden, M.D.
WHO type B3 thymoma WHO type B3 thymoma WHO type B3 thymoma

WHO type B3 thymoma

WHO type A thymomas, various patterns and atypical type A variant WHO type A thymomas, various patterns and atypical type A variant WHO type A thymomas, various patterns and atypical type A variant

WHO type A thymomas, various patterns and atypical type A variant


WHO type A thymomas, various patterns and atypical type A variant WHO type A thymomas, various patterns and atypical type A variant WHO type A thymomas, various patterns and atypical type A variant

WHO type A thymomas, various patterns and atypical type A variant

WHO type AB thymoma WHO type AB thymoma WHO type AB thymoma

WHO type AB thymoma


WHO type AB thymoma

WHO type AB thymoma

WHO type B1 thymoma WHO type B1 thymoma WHO type B1 thymoma

WHO type B1 thymoma

WHO type B2 thymoma

WHO type B2 thymoma

Thymoma: core needle biopsy

Thymoma: core needle biopsy


Thymoma: core needle biopsy Thymoma: core needle biopsy

Thymoma: core needle biopsy

Thymoma: core needle biopsy Thymoma: core needle biopsy

Thymoma: core needle biopsy

Micronodular thymoma with lymphoid stroma Micronodular thymoma with lymphoid stroma

Micronodular thymoma with lymphoid stroma


Metaplastic thymoma Metaplastic thymoma

Metaplastic thymoma

Lipofibroadenoma Lipofibroadenoma

Lipofibroadenoma

Lipofibroadenoma Lipofibroadenoma

Lipofibroadenoma


Thymoma with extensive sclerosis and ossification Thymoma with extensive sclerosis and ossification Thymoma with extensive sclerosis and ossification

Thymoma with extensive sclerosis and ossification

Small thymic epithelial cell nest

Small thymic epithelial cell nest

Cytology description
  • Dual cell population: small lymphocytes and polygonal epithelial cells (isolated or clustered); possibly sheets of bland, elongated or spindled cells or polygonal epithelial cells
  • Subtyping is not recommended
  • Reference: J Thorac Oncol 2010;5:S281
Cytology images

Contributed by Anja C. Roden, M.D.
Cytology preparation of thymoma Cytology preparation of thymoma Cytology preparation of thymoma

Cytology preparation of thymoma

Positive stains
Negative stains
Molecular / cytogenetics description
Features to report
  • Report of resection specimen should include:
    • Procedure
    • Morphology (2021 WHO classification); all components with percentages (in 10% increments)
    • Margin status
    • Extent of invasion
    • Treatment effect if applicable
    • Pleural or pericardial implants if applicable
    • Metastases if applicable
    • Lymph node status
    • Pathologic tumor stage (pTNM stage; optional in addition: Modified Masaoka Stage)
    • Background thymic gland if present
Sample pathology report
  • Thymus, thymectomy:
    • WHO type B1 invading into lung (see synoptic report)
    • The surgical margins are negative by < 0.1 cm. Multiple (6) regional lymph nodes are negative.
    • Synoptic report:
      • Procedure: thymectomy
      • Tumor size:
      • Greatest dimension: 6.5 cm
      • Additional dimensions: 5.1 x 3.2 cm
      • Histologic type: type B1 thymoma
      • Transcapsular invasion: present
      • Tumor extension: tumor involves pulmonary parenchyma, left lower lobe lung
      • Margins: uninvolved by tumor
        • Distance of tumor from closest margin: < 1 mm
      • Treatment effect: no known presurgical therapy
      • Lymphovascular Invasion: not identified
      • Regional lymph nodes:
        • Number of lymph nodes involved: 0
        • Number of lymph nodes examined: 6
      • Pathologic staging (AJCC, 8th edition)
        • TNM descriptors: not applicable
        • Primary tumor: pT3
        • Regional lymph nodes: pN0
        • Distant metastasis: pMX
      • Modified Masaoka Stage: stage III
      • Additional pathologic findings: thymic follicular hyperplasia
Differential diagnosis
  • Lymphoma / leukemia (e.g., T lymphoblastic lymphoma / leukemia, small B cell lymphomas, T cell lymphomas, classic Hodgkin lymphoma):
    • No diffuse meshwork of keratin positive neoplastic cells
    • Effaced architecture in lymphoma versus lobulated architecture in thymoma
    • LMO2 expressed in neoplastic lymphoblasts of T acute lymphoblastic leukemia (T ALL) and absent in thymoma (Am J Clin Pathol 2016;145:180)
    • CD20 positive B cells in small B cell lymphomas; classic Hodgkin lymphoma can show lobulated architecture but is comprised of mixed chronic inflammation often with eosinophils and scattered large Reed-Sternberg cells and Hodgkin cells
  • Thymic carcinoma:
    • Distorted architecture with desmoplasia and often increased cytologic atypia
    • Increased Ki67 labeling index (> 14% of tumor cell nuclei staining) in a subset of thymic carcinomas but not thymoma (Hum Pathol 2015;46:17)
    • Diffuse coexpression of CD5 and CD117 in tumor cells in a subset of thymic squamous cell carcinomas but not thymomas (Diagn Pathol 2015;10:210)
  • Metastatic carcinoma:
    • Distorted architecture, increased cytologic atypia
  • Benign thymic gland (e.g., thymic follicular hyperplasia, true thymic hyperplasia, sampling bias):
    • Lacks lobulated architecture and lacks the diffuse meshwork of keratin positive neoplastic cells (although there are keratin positive epithelial cells scattered throughout the benign thymic gland)
    • Usually intimately associated with adipose tissue without capsule
    • Lymphoid follicles in thymic follicular hyperplasia (caveat: can also be seen in micronodular thymoma with lymphoid stroma or on rare occasions in thymoma)
  • Thymic cyst:
    • Cyst with adjacent benign thymic parenchyma with or without chronic inflammation
  • Germ cell tumor (e.g., seminoma):
    • Usually morphology is different; however, seminoma can have lymphoid hyperplasia, fibrous bands and remnant thymic tissue
    • Immunostains are important
  • Thymic carcinoid tumor:
    • Might have morphologic similarities with type A thymoma, expresses neuroendocrine markers
  • Solitary fibrous tumor:
    • Might have morphologic similarities with type A thymoma; expresses STAT6, CD34
Board review style question #1

What is the neoplastic cell in this tumor?

  1. Epithelial cell
  2. Histiocyte
  3. Lymphocyte
  4. Reed-Sternberg cell
  5. Thymocyte
Board review style answer #1
A. Epithelial cell. In thymomas, epithelial cells are the neoplastic cells.

(B) Incorrect. A few scattered benign histiocytes might be present in these tumors. (C) Incorrect. Lymphocytes, which are predominantly thymocytes, are considered reactive. (D) Incorrect. Although classic Hodgkin lymphomas can exhibit a lobulated architecture, usually they do not have so sharply demarcated fibrous bands. Reed-Sternberg cells or Hodgkin cells are not identified. (E) Incorrect. Thymocytes are considered reactive in these tumors.

Comment Here

Reference: Thymoma
Board review style question #2
What is an important prognostic parameter in thymomas?

  1. Association with a cyst
  2. Male sex
  3. Presence of fibrous bands
  4. Surgical resection margin
  5. Thymic follicular hyperplasia
Board review style answer #2
D. Surgical resection margin. Complete resection is an important prognostic parameter in thymomas.

(A) Incorrect. Cystic changes have no impact on prognosis. (B) Incorrect. Sex is not associated with prognosis. (C) Incorrect. The presence of fibrous bands is not associated with prognosis. (E) Incorrect. Thymic follicular hyperplasia in the background thymic gland has no prognostic implication.

Comment Here

Reference: Thymoma

True thymic hyperplasia
Definition / general
  • Defined as an increase of thymic gland size and weight while maintaining normal microscopic architecture
Essential features
Terminology
  • True thymic hyperplasia
ICD coding
  • ICD-10: E32.0 - persistent hyperplasia of thymus
Epidemiology
  • Most often described in infants, children and young male patients
Sites
  • Anterior mediastinum
Pathophysiology
  • It is found to be the result of congenital neuroendocrine disturbances, usually within the hypothalamic hypophyseal system (Arkh Patol 1991;53:3)
Etiology
  • Underlying pathogenesis of true thymic hyperplasia is largely unknown
Clinical features
  • No symptoms, with the incidental finding of an anterior mediastinal mass on chest radiograph
  • Compression of adjacent structures: respiratory distress, cough, wheezing, superior vena cava syndrome
  • Acute or recurrent pulmonary infection
  • Peripheral blood lymphocytosis
  • References: Ann Thorac Surg 1989;47:741, J Vasc Nurs 2007;25:2
Diagnosis
  • Diagnosis is based on the demonstration of an increased thymic weight with normal histology (Connolly: Autopsy Pathology - A Manual and Atlas, 3rd Edition, 2015, Arch Pathol 1962;74:244)

    Approximate thymus weight by age (mean ± standard deviation)*
    1 month Male: 7.8 ± 5.3 g; female: 6.6 ± 4.9 g
    12 months Male: 12 ± 5 g; female: 11 ± 8 g
    6 - 25 years 25 g
    26 - 35 years 20 g
    36 - 65 years 16 g
    > 65 years 6 g
    *Values from more recent studies show great variability depending on patient populations
Laboratory
Radiology description
Radiology images

Images hosted on other servers:

25 year old woman with pure red cell aplasia

Chest CT scan showing anterior mediastinal mass

Prognostic factors
  • Overall, true thymic hyperplasia has a good prognosis
Case reports
Treatment
  • Asymptomatic patients can be followed up periodically
  • Main treatment for massive thymic hyperplasia is surgical resection
Gross description
  • Enlarged thymus gland with no mass or lesion
Gross images

Images hosted on other servers:

Intraoperative images following successful resection

Microscopic (histologic) description
  • Expansion of the normal component of the thymic gland (Histopathology 2009;54:69)
  • Normal cortex, medulla and Hassall corpuscles
  • No neoplasm
  • No increased reactive germinal centers (Histopathology 2009;54:69)
  • Lobular architecture and corticomedullary junction are preserved
Microscopic (histologic) images

Contributed by Azadeh Esmaeili, M.D. and Sarmad H. Jassim, M.D.
Expansion of normal component of thymic gland

Expansion of normal component of thymic gland

Hassall corpuscles

Hassall corpuscles

Positive stains
  • Thymocytes: TdT, CD1a, CD99
  • CD3, CD4, CD8 highlight abundant T cells; CD4 / CD8 coexpression with trailing expression of either CD4 or CD8
  • Ki67 can be used to demonstrate active thymopoiesis
  • CD20 highlights B cells
Negative stains
Sample pathology report
  • Thymus, thymectomy:
    • True thymic hyperplasia (see comment)
    • Comment: Sections show thymic tissue, characterized by lobular architecture featuring small cortical thymocytes that border interconnecting medullary tissue containing many Hassall corpuscles. The lobules are separated by mature fibroadipose tissue. There is no evidence of lymphoid hyperplasia. The average weight of the thymus gland in this age group is ~25 - 35 g. The thymus in this case weighs 90 g. These findings are consistent with true thymic hyperplasia.
Differential diagnosis
  • Thymic follicular hyperplasia:
    • Usually of normal size and weight; increased number of lymphoid follicles with germinal centers seen on histological analysis
  • Thymoma, especially the WHO type B1 thymoma:
    • Evidence of invasion, large lobules separated by fibrous bands and only rare Hassall corpuscles
  • Thymic rebound hyperplasia:
    • Rapidly growing anterior mediastinal mass in a postchemotherapy patient; some but not all cases exceed normal weight limits for age
    • Histologically normal
  • Lymphoproliferative disorders involving the thymus (e.g., T lymphoblastic lymphoma / leukemia, small B cell lymphomas, T cell lymphomas, classic Hodgkin lymphoma):
    • Effaced architecture in lymphoma
  • Germ cell tumors (e.g., seminoma):
    • Usually morphology is different; however, seminoma can have lymphoid hyperplasia and remnant thymic tissue
Board review style question #1

Which of the following statements most correctly describes true thymic hyperplasia?

  1. Effacement of normal thymic architecture
  2. Expansion of the normal component of the thymic gland
  3. Increased number of reactive secondary follicles
  4. Large lobules separated by fibrous bands
Board review style answer #1
B. Expansion of the normal component of the thymic gland. Answers A and D are incorrect because effacement of normal thymic architecture and large lobules separated by fibrous bands are features of thymoma. Answer C is incorrect because reactive secondary follicles are features of thymic follicular hyperplasia.

Comment Here

Reference: True thymic hyperplasia
Board review style question #2
Which type of thymoma can resemble the morphologic features of true thymic hyperplasia?

  1. Type A
  2. Type AB
  3. Type B1
  4. Type B2
  5. Type B3
Board review style answer #2
C. Type B1. Thymoma type B1 can closely resemble true thymic hyperplasia particularly in small biopsies; however, thymoma type B1 has larger lobules, thicker fibrous capsule and septa and predominance of cortical over medullary areas. There are no or very few Hassall corpuscles in type B1 thymoma in comparison to true thymic hyperplasia. Scattered large neoplastic epithelial cells (keratin+) obscured by a diffuse, small lymphocytic background are present.

Comment Here

Reference: True thymic hyperplasia
Back to top
Recent Mediastinum Pathology books

Allen: 2015

Bardales: 2015

IARC: 2015

IARC: 2021

Mukhopadhyay: 2023

Shimosato: 2010

Suster: 2022



Find related Pathology books: cardiovascular, lung, mediastinum/serosa
Image 01 Image 02